Ευκλειδης Α 86

Page 1

ΟΚΤΟΒΡΙΟΣ ΝΟΕΜΒΡΙΟΣ ΔΕΚΕΜΒΡΙΟΣ 2012 εupίι 3,00 •

κή

Εταιρεία


ΜΑΘΗΜΑτΙΚΟ ΠΕΡΙΟΔΙΚΟ ΠΛΗΡΟΦΟΡΗΣΗΣ

Τεύχοι; 86 Οκτώβριοι;- Νοέμβριοι;- Δεκέμβριοι; 2012

,

yια το yυμνασιο

Τιμή Τειίχοv; Εvιιώ 3,00

e-mail: info@hms.gr,

ευκλείδης

www.hms.gr

ΠΕΡΙΕΧΟΜΕΝΑ Γενικά "'Αρθρα

./ Το Παράρτημα Ν. Μαγνησίας της Ε.Μ.Ε.: Η Δράση του το 2011-2012,

./ Το εξώφυλλό μας,

./

./

.

.

.

.

.

.

.

.

.

.

.

.

.

.

.

.

.

.

.

.

.

.

.

.

.

.

.

.

.

.

.

.

.

.

.

Π. Νικολουδάκης και Σπ.Δημόπουλος

.

./ Μαθημσπκές Σχέσεις,

.

.

.

.

.

.

.

.

.

.

.

.

.

.

.

.

.

.

.

.

.

.

.

.

.

.

.

.

.

.

.

.

.

.

.

.

.

.

.

.

.

.

.

.

.

.

.

.

.

.

.

.

.

.

.

.

.

.

.

.

.

.

.

.

.

.

.

.

.

.

.

..

. . .. .. 3 ...................... 4 .

.

.

.

.

.

.

.

.

.

.

.

.

.

.

.

.

.

Ομοιότηm, Π. Νικολουδάκης και Σπ.Δημόπουλος Σελίδες για 6λοuς .

.

.

.

.

.

.

.

.

.

.

.

.

.

.

.

.

.

.

.

.

.

.

.

1

.

7

10

. . . . . . . . . . . . . . . . . . . . . 13 . . . . . 16 .

.

.

.

.

.

.

.

.

.

.

.

.

.

.

.

.

.

.

.

.

.

.

.

.

.

.

.

.

.

.

.

.

.

.

.

.

.

.

.

.

.

.

.

.

.

.

.

.

.

.

.

.

.

.

.

.

.

.

.

.

.

.

.

.

.

.

.

.

.

.

.

.

.

.

.

.

.

.

Γ"Τlιξη

Π. Νικολουδάκης και Σπ.Δημόπουλος

./ Μαθηματικοί Διαγωνισμοί,

.

.

.

./

.

Ν.Σακαλάκη

Β" Τάξη ./ Σχέδιο μαθήματος στο Πυθαyόρειο Θεώρημα, ι. Φόβος ./ Τριγωνομετρία, Π. Νικολουδάκης και Σπ.Δημόπουλος

./

.

.

Α� Τάξη Ιστορία της Αριθμητικής, Γ.Ωραιόπουλος

./ Συμμετρία,

./

.

.

./ Η

./

.

Νηλ Άρμστρονγκ ο ήρωας του Διαστήματος, κ. Μαυρομμάτης Τα Μαθηματικά και ο Πολιτισμός, Γ.Ωραιόπουλος .. . . . . . . . . . . . . . . . . . . . . . . . . . . . . . . . .

./ Το Ευπαλίνειο Όρυγμα. Ο Υδραγωγός της Σάμου,

./

.

Επιμέλεια: Σπ.Δημόπουλος

.

.

.

.

.

.

.

.

.

.

. . . . . . . . . . . . . . . . 19 23 26 28

. . . . . . . . . . ... . . . . . . . . . . . . . . .. . . . . . . 31 . 39 1ος Τσιι.ός Διαγωνισμός για μαθητές ΑΤυμvασίου, Π. Νικολουδάκης, Σπ.Δημόπουλος, Α. Κοvτοvίvα Νέες Τqνολογίες και Μαθηματικά, Στ.Μακρής .............................................. 4 1 Χαίρομαι να λύνω, Ε. Τριανταφύλλου .. . . . . . . . . . . . . . . . . . . . . . . . . . . . . . . . . . . . . . . . . . . . . . . . . . . . . . 44 Τα Μαθηματικά μας διασκεδάζουν, Σ. Γεωργίου . . . . ... . . . 48 Επιτροπή Διαγωvισμώv

.

.

.

.

.

.

.

.

.

.

.

.

.

.

.

.

.

.

.

.

.

.

.

.

.

.

.

.

.

••••••••••••••••••••••••••••••••••••••••••••••••••••••••••••••••••••••••••••••

ΕΚΔΟΣΗ 1ΗΣ ΜΑθΗΜΑτΙΚΗΣ ΕτΑΙΡΕΙΑΣ

Συντακτική επιτροπή

IWιiiERΣIIM80Y 34 106 79 ΑθΗΝΑ Τηl.: 210 3617784 3616532

Επίτιμος Πρόεδρος:

Fu: 210 3641025

Πρόεδρος: Βαρόποuλος Δήμος

Εκδότης: Καλοyερ6πουλος Γρηγόριος Διεuθuντι'jς: Εμμανουήλ Κρητικός

Αντιπρόεδρος Α": Κuρ6νας Παναγιώτης

Κωδικός ΕΛ.ΤΑ.: 2054

ISSN:

1105

-

Κιούφτη Ροϊδούλα Κuρ6νας Παναγιώτης Λαγός Γεώργιος Λuμπερόποuλος Γεώργιος Μενδωνίδης Γεώργιος Μορφοπούλοu Μαρία Μπακ6λης Αναστ6οιος Πανοuο6κης Νικόλαος Παπασταuρίδης Σταύρος Ποuλ6κη Μαρία Ρίζος Γεώργιος Σ6λαρης Κωνσταντίνος Σίοκοu Μαρία Τοαπακίδης Γεώργιος Τοικοπούλοu Στ6μη Χριστοδούλου Ντόρα Χρuοοβέργης Μιχαήλ

Ωραιόποuλος Γεώργιος

Αντιπρόεδρος Β": Λuμπερόποuλος Γεώργιος Γραμματεία: Αλαφ6κη Σταυρούλα Σίσκοu Μαρία

7998

Επιμέλεια ·εκδοαης:

Αλαφ6κη Σταυρούλα Βαρόποuλος Δήμος

Η έγκαιρη πληρωμή της συνδρομής

βοηθάει στην έκδοση του περιοδικού

Μέλη: Αγγέλη �ννα Αλαφ6κη Σταυρούλα Αλεξανδρ6τοu �ννα Αργύρη Παναγιώτα Γεωργίου Σπύρος Γληνού Αικατερίνη

Ωραιόποuλος Γεώργιος

••••••••••••••••••••••••••••••••••••••••••••••••••••••••••••••••••••••••••••••

lil/OΚfHΣIA mς ΕΜΗΝΙΚΗΣ ΜΑΘΗΜΑΠΚΗΣ ΠΑΙΡΕΙΑΣ Ετοιχειοθεtιία - Εελιδοποίηtιη: EMHNIKH Μι\ΘΗΜΙ\ΠΚΗ ΠΑ/ΡΕ/Α ROTOPRINT 14. ΜΠΡΟΥΣΜΗ & Σ/Α EEJ. τηλ.: 210 6623778 • 358

Εκτύπωαη:

Yncuθuvoς · τυnογpαφcίου: Δ. Παπαδόπουλος

• •

Τα διαφημιζόμενα βιβλία δε σημαίνει όn προτείvονrαι από την Ε.Μ.Ε. Οι συνερyασίες, τα άρθρα, οι προτεινόμενες ασκήσεις, οι λύσεις ασκήσεων κτλ. πρέπει να οτέΑvονrαι έγκαιρα, yραφεία της Ε.Μ.Ε. με την tvδειξη "Για τον Ευκλείδη Κ". Τα χειρόyραφα δεν εmοτρέφονrαι. Όλα τα ά υπόκεινrαι σε κρίση

οτα

-

lbΦ"41%!

·!Λ*Μ

Ετήσι α σuvδ(!ομή (10,00+2,00 Ταχυδ(!ομικά=εv(!ώ 12,00). Ετήσια σuvδ(!ομή για Σχολεία εv(!ώ 10,00 Το ανrίπμο yια τα τεύχη παραyyέΑvονrαι οτεΛvεται: 1. Με απλή ταχυδρομική εmταyή σε διαταyή Ε.Μ.Ε. Ταχ. Γραφείο Αθήνα 54 Τ.θ. 30044 2. Στην ιστοσεΑίδα της Ε.Μ.Ε., όπου υπάρχει δυvατ6ιητα τραπεζικής συvαλλαyής με την τράπεζα EUROBANK 3. Πληρώνεται yραφεία της Ε.Μ.Ε. 4. Με ανηκαrαβολή, σε εταιρεία ταχυμεταφορών ' χώρο σας, κατά την παραλαβή.

1100

στο

οτα


Για τη δημιουργία αυτού του τεύχους συνεργαστήκαμε με το Παράρτημα Μαγνησίας. Ευ­ χαριστούμε τους συναδέλφους για την πληθώρα ύλης που στείλανε. Τα άρθρα που δεν δημο­ σιεύονται σε αυτό το τεύχος προγραμματίζονται για επόμενα τεύχη.

Το Παράρτημα

Ν.

Μαγνησίας της Ε.Μ.Ε.: Η Δράση του το 2011-2012 Επιμέλεια: Σπύρος Δημόπουλος

Το Παράρτημα Ν. Μαγνησίας της Ε.Μ.Ε. με έδρα τον Βόλο ιδρύθηκε το 1986 και έχει ως μέ­ λη του τα μέλη της Ελληνικής Μαθηματικής Εταιρείας που ζουν ή εργάζονται στο Νομό Μαγνη­ σίας. Έχει περίπου 320 μέλη πτυχιούχους Μαθηματικούς ως τακτικά μέλη και πτυχιούχους θετι­ κών επιστημών και τεχνολογίας ως αντεπιστέλλοντα μέλη, οι οποίοι εργάζονται ως καθηγητές σε Σχολεία της Δευτεροβάθμιας εκπαίδευσης, σε Ανώτατα Τεχνολογικά Εκπαιδευτικά Ιδρύματα, στο Πανεπιστήμιο Θεσσαλίας, σε Ιδιωτικά Φροντιστήρια, ως Ιδιωτικοί Εκπαιδευτικοί, σε μεγά­ λους Δημόσιους Οργανισμούς και Υπηρεσίες του Δημοσίου και των ΔΕΚΟ, αλλά και σε ιδιωτι­ κές επιχειρήσεις. Το Παράρτημα Ν. Μαγνησίας της Ε.Μ.Ε. διοικείτε από 7-μελή Διοικούσα Επιτροπή που ε­ κλέγεται από τα μέλη του Παραρτήματος, με διετή θητεία. Η παρούσα Διοικούσα Επιτροπή προήλθε από τις εκλογές που έγιναν στις 3 Μαρτίου 2012 και της σύνθεσή της Δ.Ε. είναι η εξής: Πρόεδρος Νικολουδάκης Πέτρος, Αντιπρόεδρος Α' Καρανίκας Θωμάς, Αντιπρόεδρος Β' Κοντονινα Αναστασία, Γενικός Γραμματέας Κομνος Κώστας, Ταμίας Δημόπουλος Σπύρος, Ει�κός Γραμματέας Στάμου Ελένη, Έφορος Βιβλιοθήκης Ζιώγας Χρήστος. Σε όλη τη διάρκεια του χρόνου πραγματοποιούνται επιστημονικές, επιμορφωτικές και κοινωνι­ κές εκδηλώσεις όπως Μαθηματικοί Διαγωνισμοί, Ημερίδες Μαθηματικών, Διαλέξεις, Σεμινάρι­ α, Κοπή Βασιλόπιτας, προετοιμασία μαθητών ενόψει των Μαθηματικών διαγωνισμών, Βράβευ­ ση Μαθητών, Αποκριάτικος Χορός, Εκδρομές κ.ά. Κορυφαία εκδήλωση και δράση για το Παράρτημα Μαγνησίας τη Ε.Μ.Ε. ήταν η διοργάνωση και πραγματοποίηση στο Βόλο του 25ου Πανελληνίου Συνεδρίου της ΕΜΕ τον Νοέμβριο του 2008, με θέμα «Η Μαθηματική Εκπαίδευση και η σύνθετη πραγματικότητα του 2 1 ου αιώνα». Το Δεκέμβριο 2011 εξέδωσε ένα κομψό 48σέλιδο βιβλιαράκι με αναφορά στη Δράση του Παρατήματος κατά την τελευταία 1Οετία 2002-2011. Στην 'ιστοσελίδα του Παρατήματος: www.hms-volos.gr μπορεί να βρει κανείς όλες τις δρα­ στηριότητες μας, αναλυτικά με πλούσιο φωτογραφικό υλικό. Με βασικό γνώμονα ότι το Παράρτημά μας αποτελεί τμήμα και προέκταση της Ελληνικής Μαθηματικής Εταιρείας, ως πρώτο και βασικό στόχο έχουμε την υλοποίηση και υποστήριξη των δράσεων και αποφάσεων του Δ.Σ. της Ε.Μ.Ε. Στη συνέχεια και σε αρμονία πάντα με την κε­ ντρική διοίκηση, οργανώνουμε και τοπικές δράσεις και εκδηλώσεις, στα πλαίσια του καταστατι­ κού ΕΜΕ και επιστημονικού μας ρόλου. Ενδεικτικά αναφέρουμε τις εξής εκδηλώσεις και δράσεις του Παραρτήματος Μαγνησίας της ΕΜΕ, κατά την προηγούμενη χρονιά 2011-2012. Α) Διοργανώσαμε και πραγματοποιήσαμε τον 72° Πανελλήνιο Μαθητικό Διαγωνισμό «0 Θαλής», στην Μαγνησία. Παράλληλα πραγματοποιήσαμε και τον Τοπικό Διαγωνισμό «0 Θαλής για την Α'», για τους μαθητές και μαθήτριες της Α' Γυμνασίου, ως συνέχεια του Διαγωνισμού «Παιχνίδι και Μαθηματικά». Η συμμετοχή των παιδιών ήταν αρκετά ικανοποιητική, θα λέγαμε αρκετά μεγάλη. Το ίδιο έyινε και με την Διαγωνισμό «0 Ευκλείδηφ. Πραγματοποιήσαμε και για την Α' Γυμνασίου Διαγωνισμό «0 Ευκλείδης για την Α'». ΕΥΚΛΕΙΔΗΣ Α' 86 τ.2/1


------

Το Παράρτη μα Ν. Μαγνησίας της Ε.Μ.Ε.: Η Δράση του το 2011-2012

------

Στο διάστημα μεταξύ των Διαγωνισμών Θαλή και Ευκλείδη, πραγματοποιήσαμε μαθήματα για τους μαθητές που διακρίθηκαν. Τα μαθήματα έγιναν Σάββατα και κατά τις ημέρες των δια­ κοπών των Χριστουγέννων. Τα παρακολούθησαν περίπου 70 μαθητές. Τέλος, στις 20 Ιουνίου 2012, πραγματοποιήσαμε την Τελετή βράβευσης των μαθητών που δια­ κρίθηκαν στους Διαγωνισμούς, «ο Θαλήρ> και «0 Ευκλείδης». Στην εκδήλωση αυτή ο κ. Θρα­ σύβουλος Μαχαίρας, φυσικός, παρουσίασε μια καταπληκτική οπτικοακουστική σύνθεση - ομι­ λία με θέμα: «Οι 8 σπόνδυλοι του κόσμου». Η συμμετοχή του κόσμου ήταν κάτι παραπάνω από μαζική. Η αίθουσα που χωρούσε 350 άτομα αποδείχτηκε πολύ μικρή!! Όπως κάθε χρόνο έτσι και φέτος οι εκδηλώσεις Τελετής βράβευσης του Παραρτήματός μας, αποτέλεσαν μεγάλο πολι­ τιστικό και κοινωνικό γεγονός. Οι σχετικές φωτογραφίες το επιβεβαιώνουν. Β) Ο Διαγωνισμός «Παιχνίδι και Μ αθη ματικά» για μαθητές Ε' και ΣΤ' Δη μοτικού. Στη Μαγνησία από την πρώτη στιγμή αγκαλιάστηκε ο Διαγωνισμός τόσο από τη Δ.Ε. του Παραρτήματος όσο και από τους συναδέλφους Εκπαιδευτικούς της Πρωτοβάθμιας Εκπαίδευσης, Προϊσταμένους, Διευθυντές Σχολείων, Δασκάλους αλλά το κυριότερο από τους μαθητές και τους γονείς των. Έτσι ο 6°ς Πανελλήνιος Μαθητικός Διαγωνισμός «Παιχνίδι και Μαθηματικά» που πραγματοποιήθηκε στις 30 Μαρτίου 2012, είχε από κάθε άποψη μεγάλη επιτυχία. Στα πλαίσια σωστής προετοιμασίας, συνεργαστήκαμε με τον Διευθυντή Πρωτ/θμιας Εκπαίδευσης, με τους Σχολικούς Συμβούλους Π.Ε. και επικοινω­ νήσαμε με όλα τα Δημοτικά σχολεία του Νομού Μαγνησίας. Πραγματοποιήσαμε Ημερίδα ενημέρωσης και επιμόρφωσης των Δασκάλων που δίδασκαν στις τάξεις Ε' και ΣΓ δημοτικού σε συνεργασία με τον καθηγητή του Παιδαγωγικού Τμήματος Δημοτικής Εκπαίδευσης του Πανεπιστημίου Θεσσαλίας κ. Τριαντάφυλλο Τριανταφυλλίδη, Μα­ θηματικό και τους κ. κ Σχολικούς Συμβούλους Πρωτοβάθμιας Εκπαίδευσης Μαγνησίας. Οι συνεχείς ανακοινώσεις στον τοπικό τύπο, οι εμφανίσεις στα τοπικά τηλεοπτικά κανάλια\και ραδιόφωνα, είχαν ως αποτέλεσμα να συμμετάσχουν στον Διαγωνισμό περισσότερα παιδιά από κάθε άλλη φορά. Στο Νομό μας, με ελεύθερη συμμετοχή, έλαβαν μέρος 2087 μαθήτριες και μαθητές από 68 σχολεία, 1089 μαθητές Ε' τάξης και 998 μαθητές ΣΓ τάξης, από 170 τμήματα συνολικά. Τα γραπτά βαθμολογήθηκαν, τα περισσότερα, από τους Δασκάλους και τις Δασκάλες των παιδιών, σε πρώτη βαθμολόγηση και σε δεύτερη από Μαθηματικούς μέλη του Παρατήματος. Στις 1Ο Ιουνίου 2012, έγινε η Τελετή Βράβευσης των παιδιών που διακρίθηκαν στο Πνευμα­ τικό Κέντρο την Μητρόπολης Δημητριάδος και παραβρέθηκαν οι μαθητές και μαθήτριες που βραβεύθηκαν, οι γονείς και κηδεμόνες , συγγενείς και φίλοι, Δάσκαλοι, Διευθυντές των Σχολεί­ ων κ.ά. Γ) Το Καλοκαιρ ινό Σχολείο. Πραγματοποιήσαμε το 2° Καλοκαιρινό Μαθηματικό Σχολείο για μαθητές Ε ' και Στ' Δημοτι­ κού, που διακρίθηκαν ή βραβεύτηκαν στον 6° Διαγωνισμό «Παιχνίδι και Μαθηματικά». Τα μα­ θήματα έγιναν στο διδακτήριο του 14ου Δημοτικού Σχολείου Βόλου, από 25 μέχρι και 29 Ιουνίου 2012, δηλαδή 5 ημέρες. Το σχολείο παρακολούθησαν περίπου 180μαθητές και μαθήτριες από όλα τα σχολεία του νομού, δίδαξαν περίπου 20 συνάδελφοι και ήταν μια προσφορά των μελών του Παραρτήματος προς τα παιδιά. ΕΥΚΛΕΙΔΗΣ Α' 86 τ.2/2


Το

εξώφυλλό

μας

ντε Κίρικο,Αvήσv;ιιεςΜούσες, 1918, λάδι σε μουσαμά. Ο Τζόρτζιο ντε Κίρικο (10 Ιουλίου 1888 - 20 Νοεμβρίου 1978) ήταν Ιταλός ζωγράφος, συγγραφέας και γλύπτης Ο ντε Κίρικο γεννήθηκε στο Βόλο και ήταν ο πρωτότοκος γιος του Εβαρίστο και της Τζέμα ντε Κίρικο. Οι πρόγονοί του ήταν ιταλικής καταγωγής που εγκαταστάθηκαν και κατοικούσαν στην ανατολική Μεσόγειο αρκετές γενιές πριν. Ο πατέρας του εργαζόταν ως μηχανικός και επέβλεπε την κατασκευή του θεσσαλικού σιδηροδρομικού δικτύου το 1881, ενώ η μητέρα του ήταν πρώην τραγουδίστρια της όπερας. Η οικογένειά του εγκαταστάθηκε μόνιμα στην Ελλάδα το 1897, εννέα χρόνια μετά τη γέννηση του. Υπήρξε ο κύριος εκπρόσωπος της μεταφυσικής ζωγραφικής* και γνωστός για τη μεταφυσική και ονειρική αίσθηση του έργου του, την οποία όμως εγκατέλειψε το 1919, για να επιστρέψει σε ένα είδος κλασικισμού με επιρροές από τον Ραφαήλ, τον Ρούμπενς κ.ά. Επηρεάστηκε όχι μόνο από την κλασική αρχαιότητα, αλλά και από τη γερμανική φιλοσοφία του Σοπενχάουερ και του Νίτσε. Θεωρήθηκε ο μεγάλος πρόδρομος του σουρεαλισμού. Σπούδασε ζωγραφική στο Μόναχο από το 1906 έως το 1909. Στη διάρκεια ενός ταξιδιού του στο Παρίσι (1911-1914) γνωρίστηκε με τον Απολιναίρ και τον Πικάσο. Απέναντι στις πρωτοπορίες των αρχών του 20ού αιώνα και στις στιλιστικές εικονοκλαστικές αναζητήσεις των καλλιτεχνών του Παρισιού, ο Ντε Κίρικο προασπίστηκε μια ζωγραφική αναπαραστατική. Αντλούσε τα θέματά του από τους αρχαίους μύθους, από φαντασίες και οράματα της μνήμης. Αφοσιώθηκε στις τοπιογραφίες, στις νεκρές φύσεις και σε θέματα σχετικά με την αρχαιότητα και τον μεσογειακό χώρο. Ανάμεσα στα έργα του είναι και τα ακόλουθα: «Αίνιγμα ενός φθινοπωρινού aπομεσήμερου», «Μεταφυσικά εσωτερικά», «Μυστηριώδη λουτρά», «'Έπιπλα στην κοιλάδα», <<Έκτωρ και Ανδρομάχη», « ο μάντης». Στο παρισινό περιβάλλον ο De Chirico δείχνει να έχει ενεργοποιήσει όλο το φάσμα των δυνατοτήτων που του παρέσχε η Μνημοσύνη για να θεωρητικοποιήσει την «ποιητική» και «ηθική» διάσταση της Αντικειμενικότητας. Εκείνη την περίοδο, όσο καιρό έμεινε στο Παρίσι, δημιούργησε πλήθος σημαντικών έργων που δείχνουν τη μετάβασή του σε περισσότερες σύνθετες «θεάσεις /θεωρίες» των πραγμάτων,εισάγοντας με έναν μυστηριακό αλλά και Ύεωμετρικό Iμαθηματικό τρόπο πότε το βλέμμα του μάντη και πότε τον εμπνευσμένο λόγο του ποιητή στη σκηνή. Στο έργο του De Chiήco: αντί σε ένα «μάτι» να εστιάζεται το σύνολο των γραμμών του οπτικού πεδίου, ικανοποιώντας την απαίτηση του αναγεννησιακού ανθρώπου να ελέγξει απολύτως λογικά τον ορατό κόσμο: οι όψεις και οι κατόψεις των πραγμάτων, οι τομές και οι κλίσεις, οι βυθίσεις και οι ανώσεις των επιφανειών, οι aξονομετρικές και οι ισομετρικές προβολές, προβάλλουν τις ίδιες τις δυνάμεις των πραγμάτων. Άλλωστε προτροπή του De Chirico ήταν «να ανακαλύψουμε ένα μάτι σε κάθε πράγμα». * Με αυτόν τον όρο εννοούμε την αναπαράσταση μιας πραγματικότητας φαντασιακής, που αποτελεί πολλές φορές μια έντονη καταγγελία της απώλειας των βασικών νοημάτων της ανθρώπινης ύπαρξης. Τα έργα της μεταφυσικής ζωγραφικής έχουν αινιγματική ατμόσφαιρα. ΕΥΚΛΕΙΔΗΣ Α' 86 τ.2/3


Νηλ Άρμστρονγκ ο ηρωας του Διαστήματος ,

Του Κωνσταντίνου Δ. Μαυρομμάτη Μαθηματικού Προέδρου της Εταιρείας Αστρονομίας και Διαστήματος Στις 26 Αυγούστου 2012 αναγγέλθηκε από τα μέσα επι­ κοινωνίας ο θάνατος, από καρδιακό επεισόδιο του Αμε­ ρικανού αστροναύτη Νηλ Άρμστρονγκ (Armstrong Neil), που πρώτος πάτησε το πόδι του στη Σελήνη, στις 20 Ιου­ λίου 1969. Επρόκειτο για την απώλεια ενός μεγάλου άνδρα, που α­ νήκει πλέον σε όλη την ανθρωπότητα, μια και ο διαστη­ μικός χώρος και ειδικότερα η Σελήνη, στην οποία πρώτος περπάτησε, είναι δορυφόρος της Γης και επομένως ανήκει σε όλο τον κόσμο. Ο Νηλ Άρμστρονγκ είναι ο aστροναύτης, που έχει όλα τα χαρακτηριστικά του ήρωα, διότι και αυ­ τός ριψοκινδύνευσε τη ζωή του κατακτώντας το υπερπέραν και αψήφησε τα πάντα για να πατήσει το πόδι του στο Φεγγάρι και να προσφέρει στην ανθρωπότητα, αυτή τη μοναδική εμπειρία.

Χαρακτηριστικά ο Αμερικανός πρόεδρος Μπαράκ: Ομπάμα, όταν πληροφορήθηκε το θάνα­ τό του είπε «0 Νηλ ήταν ένας από τους μεγαλύτε­ ρους Αμερικανικούς ήρωες, όχι μόνο για την ση­ μερινή εποχή, αλλά όλων των εποχών». Ο ίδιος,

με τους άλλους δύο συναδέλφους του αστροναύ­ τες, το Έντουιν Ώλντριν (Edwin Aldrin), που τον ακολούθησε στην επιφάνεια της Σελήνης και τον Μιχαήλ Κόλινς, (Michael Collins), που περιφερό­ ταν γύρω από τη Σελήνη, απογειώθηκε με το «Α­ πόλλων - 11» (Apollo - 11), στις 16 Ιουλίου 1969, οπότε ξεκίνησε η μεγάλη αποστολή. «Σή­ μερα, το πνεύμα της αποστολής του Νηλ συνεχίζει να εμπνέει όλους τους άνδρες και τις γυναίκες που έχουν αφιερώσει τη ζωή τους για να εξερευνήσουν Οι τρεις αστροναύτες του "Απ6λλων- 11": το άγνωστο, συμπεριλαμβανομένων εκείνων που Κ6λλινς. Άρμστρονγκ. Ώλντριν. μας κάνουν να πάμε ακόμη ψηλότερα και μακρύτερα στο Διάστημα», συνεχίζει ο πρόεδρος Ομπάμα στο μήνυμά του.

Για να γνωρίσουμε καλύτερα τον Νη λ Άρμστρονγκ Ας δούμε, λοιπόν, μερικά από τη ζωή και την αστροναυτική δράση του Νηλ Άρμστρονγκ:, ο οποίος ήταν ο 25°ς άνθρωπος που εκτοξεύθηκε στο Διάστημα. Ο Νηλ Ώλντεν Άρμστρονγκ: (Neil Alden Anηstrong), γεννήθηκε κοντά στο Wapakoneta του Οχάιο (ΗΠΑ) στις 5 Αυγούστου 1930. Σε ηλικία 16 ετών απέκτησε την πρώτη του εκπαιδευτική άδεια πιλότου, προτού ακόμη κά­ νει μαθήματα για να πάρει άδεια οδήγησης αυτοκινήτου. Λίγο αργότερα, το 1947, αποφοίτησε από το Blume High School του Wapakoneta. Κατόπιν, πήρε υποτροφία από τον αμερικανικό στρατό, μπήκε στο Πανεπιστήμιο Purdue και ξεκίνησε τις σπουδές του ως αεροναυπηγού. Το ΕΥΚΛΕΙΔΗΣ Α' 86 τ.2/4


------

Νηλ Άρμστρο νγκ ο ήρωας του Διαστήματος

------

1949, έγινε χειριστής αεροσκάφους και το 1955, μπήκε στην Εθνική Συμβουλευτική Επιτροπή Αεροναυτικής (NACA), τη μετέπειτα NASA και εξελίχτηκε σε έναν από τους καλύτερους δοκι­ μαστές αεροσκαφών, πραγματοποιώντας πτήσεις με ταχύτητα πάνω από 6.000 χλμ. την ώρα, σε ύψος 60 χλμ., με το πυραυλοκίνητο αεροσκάφος Χ -15. Το 1962 επιλέχτηκε για το σώμα των αστροναυτών και το 1966 έκανε την παρθενική του πτήση στο Διάστημα με το διαστημόπλοιο «Δίδυμοι - 8», το οποίο εκτοξεύτηκε στις 1 6.3 . 1 966 μαζί με το αστροναύτη και Σκοτ Ντέιβιντ (Scott Daνid) και με κατάλληλους ελιγμούς πέτυχε να πλησιάσει και να συνδεθεί με ένα άλλο μη επανδρωμένο διαστημόπλοιο, το «Εϊτζίνα» ("Agena"). Ήταν η πρώτη σύνδεση διαστημοπλοίων στο Διάστημα. Η πτήση αυτή διήρκεσε ΙΟώ. 41λ. 26δ. Η δεύτερη διαστημική πτήση του, ως κυβερνήτη της αποστολής «Απόλλων- 1 1 », τον έφε­ ρε στη Σελήνη, όπου στις 2 1 .7. 1969 πάτησε πρώτος το πόδι του στην επιφάνειά της. Όταν η σεληνάκατος «Αετός» ακούμπησε στη Σελήνη ο Άρμστρογκ ανέφερε προς τη Γη: Εδώ βάση ηρεμίας. Ο «Αετός» προσεδαφίστηκε. Όταν δε λίγο αργότερα άρχισε να περπατάει πά­ νω στην επιφάνεια της Σελήνης περιέγραψε ως εξής το βάδισμά του: Η σκόνη κολλάει σε λεπτό στρώμα, σαν καρβουνόσκονη, πάνω στις σόλες και τα πλάγια των παπουτσιών μου. Μπορώ να την ανασηκώσω από το έδαφος με την άκρη των παπουτσιών μου. Προχωρώ με πολύ μικρά βήματα, ολίγων μόλις εκατοστόμετρων και βλέπω τα αποτυπώματα των παπουτσιών μου και τα ίχνη των βημάτων μου πάνω στη λεπτόκοκκο αμμώδη έκταση. Δεν νιώθω δυσκολία στις κινήσεις μου ...

Μετά από λίγο κάθισε απέναντι από την κάμερα της σεληνακάτου, ώστε να τον βλέπουν οι κάτοικοι της Γης από τις τηλεοράσεις τους και διάβασε την επιγραφή της πλάκας που ήταν στη βάση του «Αετού»: Εδώ άνθρωποι από τον πλανήτη Γη πάτησαν πρώτοι το έδαφος της Σελήνης.

Ιούλιος 1969. Ήρθαμε με προθέσεις ειρηνικές για όλη την ανθρωπότητα.

Η διάρκεια της πτήσης αυτής ήταν 8ημ. 19ώ. 1 8λ. 35δ., από την οποία οι 2 1ώ. 36λ. ήταν χρόνος παραμονής στην επιφάνεια της Σελήνης. Ο Νηλ Άρμστρονγκ παραιτήθηκε από τη NASA το 197 1 , έπειτα από θητεία 17 ετών για να γίνει καθηγητής αεροδιαστημικής μηχανικής στο Πανεπιστήμιο Σινσινάτι. Το 1979, αποσύρθηκε και από το Πανεπιστήμιο και ασχολήθηκε με ιδιωτικές εργασίες. Δεν επιθυμούσε τη δημοσιότη­ τα, δεν ήθελε διακρίσεις, ούτε εκδηλώσεις προς τιμή του. Το 1995 ήταν διευθυντής εταιρείας ΗΝ μαζί με το γιο του Έρικ. Στην πατρίδα του, στο Οχάιο, ιδρύθηκε «Μουσείο Αεροδιαστημι­ κής Νηλ Άρμστρονγκ», όπου βρίσκονται πολλά πράγματα που χρησιμοποίησε ο ίδιος, ως a­ στροναύτης. Έφυγε από τη ζωή στις 25 Αυγούστου 2012, σε ηλικία 82 ετών.

ΕΥΚΛΕΙΔΗΣ Α' 86 τ.2/5


τ.ο η μερ ολόγιο

j

li

εγχειρfι ματος

Ένα κάπως λεπτομερές ημερολόγιο με τα διαδοχικά βήματα του μεγάλου εγχειρήματος της πρώτης κα­ τάκτησης της Σελήνης έχει ως εξής, όπως περιγράφεται στο «Λεξικό Διαστημικής»:

Σάββατο, 19 Ιουλίου

7.13' μ.μ. Το διαστημόπλοιο διέρχεται πίσω από τη Σελήνη και, για πρώτη φορά, χάνεται κάθε τηλεπικοι­ νωνιακή επαφή με τη Γη. 7.28' μ.μ. Το διαστημόπλοιο εισέρχεται σε σεληνιακή τροχιά. 7.23' μ.μ. Αποκαθίστανται οι τηλεπικοινωνίες μεταξύ διαστημοπλοίου και Γης. 9.56' μ.μ. Έναρξη έγχρωμης τηλεοπτικής εκπομπής της σεληνιακής επιφάνειας προς τη Γη, διάρκειας 34ων λεπτών. 12.57' μ.μ. Οι αστροναύτες περνούν στη σεληνάκατο για επιθεώρηση. 3.17' π.μ. Οι αστροναύτες επανέρχονται στο θάλαμο διακυβέρνησης.

Κυριακή, 20 Ιουλίου

3.27' μ.μ. Ο Ώλντριν διεκπεραιώνεται στη σεληνάκατο και μετά από μια ώρα και ο Άρμστρονγκ. 7.46' μ.μ. Η σεληνάκατος «Αετός» αποχωρίζεται από το βασικό διαστημόπλοιο «Κολούμπια». Ο Άρμ­ στρονγκ αναφέρει «0 Αετός έχει δικά του φτερά». 8.12'μ.μ. Ο Κόλλινς πυροδοτεί του πυραύλους- βερνιέρους του «Κολούμπια», το οποίο έτσι κάνει ελιγ­ μούς και απομακρύνεται 3,2 χλμ. από τη σεληνάκατο, και σε διαφορετική τροχιά. 9.08' μ.μ. Οι αστροναύτες της σεληνακάτου πυροδοτούν για πρώτη φορά τον κινητήρα καθόδου. 10.05' μ.μ Αρχίζει η κάθοδος προς τη Σελήνη με τον ανασχετικό πυραυλοκινητήρα καθόδου τη στιγμή ·που η σεληνάκατος βρίσκεται σε ύψος 15.240 χλμ. Ύστερα από 13 λεπτά η σεληνάκατος προσεδαφίζεται. 10.18' μ.μ. Η σεληνάκατος «Αετός» με τους αστροναύτες Άρμστρονγκ και Ώλντριν προσεδαφίζονται στη Σελήνη. Ο Άρμστρονγκ αναφέρει: «0 Αετός προσεδαφίστηκε». 3.53'π.μ. (21 Ιουλίου). Αποσυμπιέζεται η καμπίνα του «Αετού» για να ανοίξει η θυρίδα και να c."'"'""�""v θήσει η κάθοδος στη σεληνιακή επιφάνεια. 4.39' π. μ. Ο Άρμστρονγκ αναφέρει ότι η θυρίδα απασφαλίστηκε. 4.56' π.μ. Ο Άρμστρονγκ κατεβαίνει τη σκάλα με τα 9 σκαλοπάτια, πατάει στη σεληνιακή επιφάνεια και κάνοντας το πρώτο βήμα παρατηρεί: «Αυτό είναι ένα μικρό βήμα για τον άνθρωπο, ένα γιγάντιο άλμα για την ανθρωπότητα». 5.14' π.μ. Ο Ώλντριν κατεβαίνει στη σεληνιακή επιφάνεια πηδώντας από το τελευταίο σκαλοπάτι. 5.41' π.μ. Ο Άρμστρονγκ στήνει την αμερικανική σημαία στην κονιορτώδη επιφάνεια. 5.48' π.μ. Ο πρόεδρος Νίξον μιλά στους Άρμστρονγκ και Ώλντριν από το Λευκό Οίκο, 402.330 χλμ. μα­ κριά από τη Σελήνη.

ευτέ α, 21 ουλίου

6.54' π.μ. Ο Ώλντριν επιστρέφει στον «Αετό», αφού τελείωσε τον σεληνιακό περίπατό του. 7.09' π.μ. Ο Άρμστρονγκ επιστρέφει και αυτός στη σεληνάκατο, αφού παρέμεινε 2 ώρες και 11 λεπτά στη σεληνιακή επιφάνεια. 7.11' π.μ. Η θυρίδα της σεληνακάτου κλείνει και αρχίζει η εκ νέου συμπίεση της καμπίνας. 7.54' π.μ. Πυροδοτείται ο κινητήρας ανόδου και ο «Αετός» ανυψώνεται, ύστερα από παραμονή στη νη επί 21 ώρες και 37 λεπτά. 11.35' μ.μ. Το διαστημόπλοιο «Κολούμπια», όπου βρίσκεται ο Κόλλινς ενώνεται με τον «Αετό», 11Ο πάνω από την αθέατη πλευρά της Σελήνης.

ρίτη, 22 Ιουλίου

6.56' π.μ. Πυροδοτείται ο κινητήρας του διαστημοπλοίου, ο οποίος το οδηγεί έξω από τη σεληνιακή με κατεύθυνση προς τη Γη. 7.39'μ.μ. Το διαστημόπλοιο σε απόσταση 62.640 χλμ. από τη Σελήνη, μεταπίπτει στο πεδίο έλξεως της Γης. 10.02' μ.μ. Οι πυραυλίσκοι ελιγμών πυροδοτούνται επί 9,5 δευτερόλεπτα για τη διόρθωση της τροχιάς. 3.08' π.μ. Προτελευταία τηλεοπτική εκπομπή, διάρκειας 18 λεπτών, δείχνει τους τρεις αστροναύτες γευματίζουν.

Τετάρτη, 23 Ιουλίου

9.56' μ.μ. Το διαστημόπλοιο διέρχεται από το μέσο της διαδρομής του ταξιδιού επιστροφής, 187.000 από το σημείο προσθαλασσώσεως. 1.03' π.μ. (24 Ιουλίου). Έναρξη της τελευταίας τηλεοπτικής εκπομπής, που δείχνει τη Γη να πλησιάζει.

Πέμπτη, 24 Ιουλίου

6.21' π.μ. Ο θάλαμος διακυβερνήσεως αποχωρίζεται από το βοηθητικό τμήμα εξυπηρετήσεως που περιλαμβάνει τον κύριο πυραυλοκινητήρα. 6.35' μ.μ. Ο θάλαμος διακυβερνήσεως εισέρχεται εκ νέου στην ατμόσφαιρα της Γης. .51' μ.μ. Το «Απόλλων- 11» προσθαλασσώνετ\χ,ι στον Ειρηνικό Ωκεανό, νοτιοδυτικά της Χαβάης. •

'-4ι

-

\

;

•Ι ι


Γ. Ωραιόπουλος

1. KINA

ΜΕΣΟΙ XPONOI ου

ΚΟΜΦΟΥΚΙΟΣ (551-475π.Χ.) Ηθικοπο­ λιτικός, φιλόσοφος και νομοθέτης, που ανέ­ πτυξε τον πνευματικό πολιτισμό της κοινωνι­ κής πολιτικής στην Κίνα για 2000 χρόνια. Πί­ στευε ότι υπάρχουν άνθρωποι εκμεταλλευτές και εκμεταλλευόμενοι. Οι πρώτοι κυβερνούν και οι δεύτεροι υποτάσσονται στους πρώτους, που τους συντηρούν με την εργασία τους. Οι ηθικοί κανόνες βασίζονται στο σεβασμό των γεροντότερων, των κυβερνητών, των γονιών. Έγραφε: «0 ηγεμόνας είναι γιος του Ουρανού, ο πατέρας του είναι πατέρας. Ο γιος είναι γι­ ος)).

Επιλύουν συστήματα 1 βαθμού με 2 και 3 αγνώστους. Ο Τσινγκ Τσου εισήγαγε τους αρνητικούς αριθμούς και πέτυχε εξαγωγή τε­ τραγωνικής και κυβικής ρίζας. Στην Αστρονομία γνώριζαν τα στοιχεία των πλανητών, τις εκλείψεις. Ο Σιχ Σεν στο βιβλίο του έχει έναν κατά­ λογο με 807 άστρα. Στη μηχανική δημιούργησαν πολλά εργα­ λεία. Ακόμα ανακάλυψαν την πυξίδα, την πυ­ ρίτιδα, τη σηροτροφία με μεταξοσκώληκες . Τον 5° αιώνα κατασκεύασαν το χαρτί και αρ­ γότερα την τυπογραφία. Στην περίοδο αυτή η Κίνα με 1,5 δισεκα­ τομμύρια κατοίκους, είναι καθυστερημένη μι­ σοαποικιακή. 2. ΙΝΔΙΑ

Οι πάνω από 300 εκατομμύρια Ινδοί σε ο­ ρισμένες περιοχές της χώρας τους ασχολήθη­ καν με τα Μαθηματικά και την Αστρονομία. Με τις κατακτήσεις του Μεγάλου Αλεξάνδρου γύρω από τον Ινδό ποταμό μερικοί στρατηγοί του ίδρυσαν μικρά βασίλεια και προσπάθησαν να φέρουν σ' αυτά τον αρχαιοελληνικό πολι­ τισμό παρά την αντίδραση των βουδιστών που τους διαδέχθηκαν στα βασίλεια. Τον 5° και 6° αιώνα ο μαθηματικός Αρια­ μπχατα έγραψε ποιήματα το «Aryabhata)) που περιείχε αριθμητικό λογισμό και ουράνια Α­ στρονομία. Επίσης ο Brahmarupa (7°ς αι.) έλυσε αριθμητικο-αλγεβρικά προβλήματα χρησιμοποιώντας το Ο και αρνητικούς αριθ­ μούς. Εξαίρετος μα­ θηματικός Bascara ( 12°ς αι.) με το βιβλίο του «Lilabato)) λύει την εξίσωση χ2+45χ250=0 με ρίζες 50 και -5. Ακόμη την τριτο­ εξίσωση βάθμια

Δεν υπάρχουν πληροφορίες για τα Μαθη­ ματικά και άλλες επιστήμες την περίοδο αυτή, η τέχνη όμως της αγγειοπλαστικής με τα περί­ φημα γεωμετρικά αγγεία μαρτυρούν ότι υπήρ­ χαν γνώσεις μαθηματικών. Οι Κινέζοι υπο­ στηρίζουν ότι ανακάλυψαν το Πυθαγόρειο Θεώρημα πριν από τον Πυθαγόρα και τον υ­ πολογισμό του π πριν από τον Αρχιμήδη. Οι ίδιοι αναφέρουν ότι ορισμένοι aυτο­ κράτορες έκαψαν συγγράμματα επιστημονικά και τους συγγραφείς τους. Από το 2000 π.Χ. εμφανίζονται βιβλία μαθηματικά με aστρονο­ μικές προσφορές. Από τα βιβλία που έγραψαν αναφέρουμε χ3+ 12χ2-6χ+35=0. το «Το πολύτιμο κάτοπτρο)) και την «Κλασική Το πιο γνωστό επίτευγμα των Ινδών στα Αριθμητική)). Μαθηματικά και τον Πολιτισμό είναι το δεκαδιΕΥΚΛΕΙΔΗΣ Α' 86 τ.2/7


-------

Τα Μαθηματικά και ο Πολιτισμός

--------­

κό σύστημα θέσης με τα 10 σύμβολα γκάμ ( 1 1 ος αιώνας) που έγραψε ένα σωστό η0,1 ,2,3,4,5,6,7,8,9 που χρησιμοποιούμε σήμερα. λιακό ημερολόγιο, που δεν εφαρμόστηκε επειδή το Ισλάμ ήθελε το σεληνιακό. 3. ΗΑΡΑΒΙΚΗΠΡΟΣΦΟΡΑ

Τον 7° αιώνα μ.Χ. εμφανίστηκαν οι Άρα­ βες στη Μέση Ανατολή, οι οποίοι επιβλήθη­ καν σε όλους τους γύρω λαούς. Προχώρησαν σ' όλη τη Βόρειο Αφρική και την Ισπανία. Με τη γλώσσα τους και τη θρησκεία, το Ισλάμ του Μωάμεθ προσπαθούσαν να αντικαταστήσουν τον Ελληνορωμαϊκό πολιτισμό με τον αραβι­ κό. Πνευματικό τους κέντρο ήταν η Βαγδάτη όπου δημιούργησαν σχολές στις οποίες μελε­ τούσαν και μετάφραζαν κυρίως αρχαιοελληνι­ κά κείμενα. Αυτή ήταν μια προσφορά στον πολιτισμό και τα Μαθηματικά, γιατί πολλά βι­ βλία χάθηκαν και έμειναν οι μεταφράσεις τους. Θα αναφέρουμε μερικά ονόματα. Ο Αλ Χουάρσμ έγραψε το μαθηματικό βιβλίο « . . . Αλ Τζαμπρ». Αυτό το μετάφρασαν οι Ευ­ ρωπαίοι στα Λατινικά, Άλγεβρα κι έτσι δημι­ ουργήθηκε ο νέος κλάδος των Μαθηματικών. Ο Αλ Χάγκα (870-920) και άλλοι μαθημα­ τικοί αστρονόμοι μετάφρασαν «Τα στοιχεία» του Ευκλείδη, τη «Μεγάλη Μαθηματική Σύ­ νταξη» του Πτολεμαίου, «Τα Αριθμητικά» του Διόφαντου. Ο Αμπού Αλ Βαφα μαθηματικός aστρονόμος στο έργο του «Βιβλίο γεωμετρι­ κών κατασκευών» γράφει προβλήματα του Ευκλείδη, του Πάππου κ.α. Στην αραβική εγκυκλοπαίδεια «Απελευθέ­ ρωση» υπάρχουν κεφάλαιο Αριθμητικής, Γε­ ωμετρίας, Αστρονομίας και Μουσικής. Στη Σχολή της Βαγδάτης εγκαταστάθηκε ο μαθηματικός Αμπού Ταμπίι (830-900) γνώ­ στης της Αρχαίας Ελληνικής Γλώσσας, έκανε πολλές μεταφράσεις των Ελληνικών Μαθημα­ τικών. Αυτός απέδειξε με νέο τρόπο το Πυθα­ γόρειο Θεώρημα. Μαζί με άλλους μελέτησαν την Τριγωνομετρία και έγραψαν πίνακες τρι­ γωνομετρικών αριθμών στους οποίους έδωσαν ονόματα, τα οποία οι μεταφραστές στη Λατι­ νική τα έγραψαν Sinus (ημ), Cosinus (συν). Ο Πέρσης μαθηματικός aστρονόμος ίδρυ­ σε στη Βαγδάτη αστεροσκοπείο και βιβλιοθή­ κη που έφερε το όνομά του Νατίρ Ιντ Ντιν. Περί το 1000 μ. Χ. έφθασε στην Περσία η φυλή Σελτζούκοι Τούρκοι, εκεί έζησε ένας άλλος aστρονόμος φιλόσοφος ο Ομάρ Χα-

Η μεγάλη αυτή μαθηματική και πολιτισμι­ κή προσφορά των αράβων κράτησε πέντε αιώ­ νες. 4. Τ Ο ΒΥΖΑΝΤΙΟ

Η Ρωμαϊκή Αυτοκρατορία με τις ανατολι­ κές κατακτήσεις της ίδρυσε νέο κράτος το Βυ­ ζάντιο τον 4° αιώνα μ.Χ. με πρωτεύουσα την Κωνσταντινούπολη και βασιλιά το Ρωμαίο Κωνσταντίνο που έγινε χριστιανός Ορθόδο­ ξος και μάλιστα άγιος με τη μητέρα του Ελέ­ νη, που γιορτάζουν κάθε χρόνο στις 21 Μαίου.

Το κράτος αυτό διατηρήθηκε περίπου 1000 χρόνια με Έλληνες aυτοκράτορες και πατριάρχες. Σε συνεργασία με Αθήνα και Α­ λεξάνδρεια δημιούργησε καλό πολιτισμό με τα Μαθηματικά και τις άλλες επιστήμες, παρά τους συνεχείς πολέμους με εχθρούς από την Ανατολή, του Βορρά και τη Δύση μέχρι το 1453 που καταχτήθηκε από τους Τούρκους. Οι σπουδαιότεροι Βυζαντινοί Μαθηματι­ κοί ήταν: Ο Λέων (9°ς αι.) μαθηματικός και θεολό­ γος που δίδαξε Μαθηματικά και Αστρονομία. Παράλληλα διατέλεσε μητροπολίτης Θεσσα­ λονίκης.

ΕΥΚΛΕΙΔΗΣ Α' 86 τ.2/8


------- Τα Μαθηματικά και ο Πολιτισμός

Ψελλός Μιχαήλ

( 1 1 ος αι.) Λόγιος, μαθηματικός και πολιτικός. Διευθυντής της Φιλοσοφικής Σχολής έγραψε: «Αριθμητική, Γεωμετρία, Μουσική και Αστρονομία». Πλανούδης Μάξιμος (12°ς αι.) μαθηματικός, Λογοτέχνης Έγραψε: «Ψηφοφορία κατ' Ινδούς» για το δεκαδικό σύστημα. Νικηφόρος Βλεμμίδης (1 3°ς αι.) Έγραψε «Επιτομή Λογικής, Φυσικής, Γεωγραφίας» και «Αριθμητική».

-------

Βοήθιος

(480-524) που έγραψε «Μαθηματι­ κά» μετάφραση της Αριθμητικής του Αλεξαν­ δρινού μαθηματικού Νικόμαχου. Ένας άλλος εκκλησιαστικός δάσκαλος του 1 σου αιώνα ή­ ταν ο Γάλλος Γκέρβερ που σπούδασε στην Ισπανία και έγινε ο Πάπας Σιλβέστρος.

.. ,. .._.... .... . ,_ .. ·�·. .,..... .,,.... ..� · --·�·..,�·... ---- · --

·.ι.� .· ·...r:..:. ;,... -τ'"... �ι (, -.... � . .!,, ...... �,&.;� ..

Η οικονομική ζωή βασιζόταν στη γεωργία, που οι δούλοι αντικαταστάθηκαν από δουλο­ ,.... ..... ·---:..�...... ......... ... πάροικους που δούλευαν σε μεγάλα τσιφλίκια - φέουδα, δηλαδή δημιουργήθηκε νέα τάξη η φεουδαρχική του Μεσαίωνα. Οι Βόρειοι Ευ­ ρωπαίοι δημιούργησαν κράτη φράγκικα, γερ­ μανικά. Παράλληλα όμως δημιουργήθηκαν εμπορικές πόλεις όπου αναπτύχθηκαν σχολεία Στη Σχολή της Τραπεζούντας δίδαξαν και πανεπιστήμια. Στην ιταλική πόλη Πίζα, Μαθηματικά και Αστρονομία οι Ισαάκ Αργυ­ όπου και ο κεκλιμένος πύργος έζησε ο μεγά­ ρός και Χιουνιάδης Γεώργιος. Οι Μαθηματι­ λος μαθηματικός ο Λεονάρδος Πιζάνος Φι­ κοί Αρχιτέκτονες Ανθέμιος Τραυλός και Ισί­ μπονάτσι (1 170-1250) ο οποίος ύστερα από δωρος Μιλήσιος έκτισε το μεγαλοπρεπή Ναό ταξίδια στην Ανατολή, έγραψε δυο βιβλία «το Λίμπερ Αβατσι» (βιβλίο Άβακα) και «Εφαρ­ -Μουσείο της Αγίας Σοφίας. μογές Γεωμετρίας». Ιστορική είναι η σειρά του:Ο, 1 , 1 ,2,3,5,8, 13 ... , όπου κάθε προηγούμε­ νος αριθμός είναι άθροισμα των δυο προηγού­ μενων. Αυτός έλυσε πολλά προβλήματα Α­ ριθμητικής, Γεωμετρίας, Άλγεβρας και Τριγω­ νομετρίας. Το π=3,141 8 1 5. "nιαι."4ur.rtu;

.� ,ι • •• • •.t. >��

,.,

"' •.L.L...U

.

. 4

...•..............

Δημιουργήθηκαν έργα τέχνης και τεχνολο­ γικά έγιναν πολλά για τη βελτίωση της ζωής των κατοίκων. 5. ΜΕΣΑΙΩΝΑΣ ΣΤΗΝ ΕΥΡΩΠΗ

Τον 5° μ.Χ. αιώνα άρχισε η παρακμή της Δυτικής Ρωμαϊκής Αυτοκρατορίας, ενώ αυξή­ θηκε ο ρόλος της καθολικής εξουσίας του Πά­ πα. Τα μοναστήρια ήταν και σχολεία, που δί­ δασκαν καλόγεροι. Ένας απ' αυτούς ήταν ο

Μέχρι τον 15° αιώνα εμφανίστηκαν και άλλοι μαθηματικοί. Στο Πανε­ πιστήμιο της Μπολώνια ο Πατσόλι, στη Σικελία ο Έλληνας Μαυρόλυκος κ. α Επίσης στο Μεσαίω­ να αναπτύχθηκε η Τέχνη με άξιους ζωγράφους, μουσικούς και η Τεχνική. Δεν είναι λοιπόν σκοτεινά τα χρόνια αυτά, έχουμε ίσως λίγο πολιτισμό και λίγα Μαθηματικά.

ΕΥΚΛΕΙΔΗΣ Α' 86 τ.2/9

•••••••••••••••••••


ΤΟ ΕΥΠΜΙΝΕΙΟ ΟΡΥΓΜΑ Ο

Υδραγωγός της Σάμου

======

Το Νίκου Ε. Σακαλάκη, Μαθηματικού, στο 1 ο Γυμνάσιο Βόλου.

Α. Ε ισαγωγή

Αναμφίβολα, μια από τις ικανότητες του ανθρώπου είναι να θέτει στην υπηρεσία των ανα­ γκών του πολύ περισσότερα στοιχεία του περιβάλλοντος απ' ότι ένα ζώο. Στοιχεία πολλές φορές ακατάλληλα γι' αυτόν στην ακατέργαστη φυσική τους κατάσταση, έ­ γιναν κατάλληλα έπειτα από την παρέμβαση της ανθρώπινης λογικότητας.

Αυτός ο ανώτερος και πληρέστερος τρόπος προσαρμογής στο περιβάλλον σε σχέση με τα ζώ­ α, αποτελεί χάρισμα του Δημιουργού Θεού στον άνθρωπο. Το Ευπαλίνειο Όρυγμα στην αρχαία Σάμο, η μεγάλη αυτή σήραγγα, που έφερνε το νερό από την πηγή στην πρωτεύουσα, αποτελεί ένας παράδειγμα της μεταβολής ακατέργαστων φυσικών στοιχείων, σε μέσα ανάπτυξης της α' ν­ θρώπινης δραστηριότητας. Στο υδραγωγείο αυτό (αμφίστομο όρυγμα) εστιάστηκε και το ιστορικό βάθος της σκέψης του Ηρόδοτου και όχι μόνο, διότι εξυμνεί και δύο άλλα μεγάλα έργα της Σάμου, που είναι : Ο ναός της Ήρας και η προκυμαία του λιμένος(το εν θαλάσση χώμα). Ο Γάλλος Freνille διατύπωσε μια σκέψη, ίσως υπερβολική : «Δεν είναι το περιεχόμενο της ι­ στορίας λογικό, αλλά το περιεχόμενο της λογικής ιστορικό» Έχω τη γνώμη, ότι ο Ηρόδοτος στο ίδιο θέμα απαντά μέσα από τις ιστορικές αναφορές του με περισσότερη ακρίβεια. Ίσως να έλεγε : «Δεν είναι το περιεχόμενο της ιστορίας παράλογο, αλλά είναι το περιεχόμενο της λογικής Ιστορικό» Το Ευπαλίνειο Όρυγμα δεν είναι στενά ένα «ιστορικό» θέμα, διότι η μαθηματική παιδεία των Ελλήνων δεν είναι απλά μια ιστορική πραγματικότητα που είχε σημασία μόνο στην εποχή της. Η Μαθηματική παιδεία των Ελλήνων είναι πάντοτε πνευματικά παρούσα. Β. Πως

βλέπει ο Η ρόδοτος τα έργα των Σαμίων - Έργα πολιτισμού. Θα μπορούσε, άραγε, ο Ηρόδοτος να περιγράψει τόσο παραστατικά τα έργα των Σαμίων, χω­ ρίς να έχει συνειδητοποιήσει βαθειά, ότι κάθε μεταμόρφωση των πραγμάτων του εξωτερικού κόσμου για ορθή ανθρώπινη χρήση είναι έργο πολιτισμού; ΕΥΚΛΕΙΔΗΣ Α' 86 τ.2/10


------

ΤΟ ΕΥΠΑΛΙΝΕΙΟ ΟΡΥΓΜΑ Ο Υδραγωγός της Σάμου

-------­

Σήμερα, βέβαια, πολλά επιτιμητικά θα έγραφε για όλα εκείνα τα έργα, που έχουν έλλειψη ζω­ ντανής πνευματικότητας και δημιουργικότητας και αποτελούν καταστροφή για τον άνθρωπο μακροπρόθεσμα. Για τα έργα των Σαμίων γράφει σχετικά: ( στο έργο του «'Ηροδότου Μοϋσαι - 'Ιστοριών τρίτη έπιγραφομένη Θάλεια -έκδοση ed. Α. D. Godley. Cambridge 1 920) : 60.

« Έμιjκυνα δέπερiΣαμίων μόΛλον, όrι σφι τρία iστίμέyιστα όπόvτων Ελλήνων έξεργασμέvα, Φεά; τε ύψηλού ές πεντήκοντα καί &ατόv φγυιό;, τούτου Φυγμα κόιωθεν όρξόμενον, όμφ ίστομον. »

Ειδικά για το όρυγμα,συνεχίζει : -

«τΟ μεv μηκος του Ορυyματος επτa στόδιοι ε/σΙ, το δε υψος και ευρος Οcτω εκarερον πόδες. δια παντά; δέαύrού όλλο Φυγμα είκοσίπηχυ βάθος ΦCΨυκται, τρlπουν δέτόεόρος, δι' οiJτό ύδωρ όχετευψενον διό τώv σωλήνων παραγίνεται ές τήv πόλιν όγψενον όπό μεγόλης πηγfj;. [3] όρχιτ8cτων δέτού φύyματος τούτου έyέvετο Μεγαρεύ; ΕύπαλίVος Ναυστράρου . . .. . » ι

ι

-

ι

r

ι

,

ι

,

ι

ι

"

ι

7'

ι

ι

r

ι

,

ι

Στη μετάφραση ο Ηρόδοτος μας λέει τα εξής : « . . . Μίλησα δε περισσότερο για τους Σαμίους, επειδή αυτοί εκτέλεσαν τρία έργα, τα οποία εί­ ναι τα μεγαλύτερα από όλα όσα εκτέλεσαν οι Έλληνες. Κάτω από όρος ψηλό έως 1 50 οργιές κα­ τασκεύασαν όρυγμα αμφίστομο. Το μήκος του ορύγματος είναι 7 στάδια, το δε ύψος και το πλά­ τος του 8 πόδια το καθένα.

Αιιιlι

. ι....ι.... ι

ι

....

I

...

ι 1 1 • ....

•• -

ι

ι ι

��-�

I

,.

ι

ι

ι

1

I

1010

����

Σ' όλο το μήκος του ορύγματος τούτου, σκάφτηκε άλλο όρυγμα βάθος 20 πήχεων και πλά­ τους 3 ποδιών, μέσω του οποίου το νερό μεγάλης πηγής φέρεται με σωλήνες στην πόλη. Αρχιτέ­ κτονας του ορύγματος τούτου ήταν ο Μεγαρεύς Ευπαλίνος, γιος του Ναυστρόφου . . . . . » Προς κατανόηση των μεγεθών που αναφέρει ο Ηρόδοτος σημειώνουμε τα εξής : Μια οργιά περίπου 1 ,50 μέτρα. Ένα στάδιο περιέχει 1 00 οργιές, δηλαδή 150 μέτρα. Μια οργιά είχε 6 πόδια, δηλαδή κάθε πόδι ήταν 0,25 μέτρα. Ένας πήχυς ήταν περίπου 0,4 1 μέτρα. •

• •

ΕΥΚΛΕΙΔΗΣ Α' 86 τ.2/11


------

ΤΟ ΕΥΠΑΛΙΝΕΙΟ ΟΡΥΓΜΑ Ο Υδραγωγός της Σ άμου

-------­

Άρα το όρος έχει ύψος περίπου 225 μέτρα, το όρυγμα έχει μήκος 1036 μέτρα και πλάτος ίδιο με το ύψος, δηλαδή 2 μέτρα. Γ . Ε πεξηγήσεις

Το Ευπαλίνειο όρυγμα έγινε με εντολή του Πολυκράτη (6°ς π.Χ. αιώνας), όταν αυτός βασί­ λευε στη Σάμο και χρειάστηκαν για την κατασκευή του 1 1 περίπου χρόνια. Σκοπός του ορύγματος ήταν η μυστική υδροδότηση πρωτεύουσας (Πυθαγόρειο), ώστε σε πε­ ρίπτωση πολιορκίας ρο δίκτυο ύδρευσης να είναι ασφαλές και αθέατο από τους εχθρούς. Η μηχανική σπουδαιότητα του έργου βρίσκεται στην ταυτόχρονη διάνοιξη του βουνού και από τις δύο αντικείμενες πλευρές εκατέρωθεν του βουνού., με αποτέλεσμα να συναντηθούν στο ίδιο σημείο με μικρή απόκλιση ολίγων εκατοστών. Μέσα στη σήραγγα (τούνελ) έγιναν και άλλα επί μέρους σπουδαία τεχνικά έργα, για την εύ­ κολη και ασφαλή ροή του νερού στην πρωτεύουσα από την πηγή καθώς και για την άνετη διέ­ λευση ανθρώπων για έλεγχο και συντήρηση του υδραγωγείου. • • •

Δ. Μαθη ματικά στοιχεία του ορύγματος.

Στο σημείο Ε το Νότιο συνεργείο έφθασε μετά από 8,5 χρόνια. Στο Δ είχε φθάσει το βόρειο συνεργείο γύρω στα 5 χρόνια Στο ΑΒΔ έγινε ισοσκελής ελιγμός, δηλαδή ΑΒ=ΒΔ και το τμήμα ΔΕ είναι η μικρή απόκλιση των δύο συνεργείων. Το επίπεδο που ορίζουν τα σημεία Α, Β, Δ είναι οριζόντιο και όχι κατακόρυφο και ο ελιγμός Α-+ Β-+ Δ έγινε για να αποφευχθούν τα σαθρά πετρώματα. Κατά τον ελιγμό σχηματίστηκε το ισοσκελές τρίγωνο ΑΒΔ με ΑΒ=ΒΔ = 1 39 μέτρα και & = }_

32

της ορθής. Ο αρχικός όμως άξονας διάνοιξης ΚΛ παρέμεινε ευθύγραμμος !! Ε

ΜΑΘΗΜΑ1ΙΚΑ ΠΟΙΧΕΙΑ ΟΡΥΓΜΑΤΟΣ • ο • I

ο • ο Σ Σ γ Ρ Α

1 ο • I

ΒΟΡΑΣ

Α Σ

Η αρχιτεκτονική του ορύγματος μας πείθει ότι ο Ευπαλίνος ήταν μεγάλος τοπογράφος, μεγά­ λος μαθηματικός και σπουδαίος πολιτικός μηχανικός. � � �

Ε. Και μερικά ερωτήματα . . .

Πώς ένα αρχαίο έργο 2500 ετών συναγωνίζεται σε ακρίβεια μετρήσεων και κατασκευής τα σύγχρονα έργα, όπως για παράδειγμα το Αττικό μετρό, όπου είχαμε αποκλίσεις της τάξεως του μέτρου ενώ στο Ευπαλίνειο όρυγμα είχαμε απόκλιση της τάξεως εκατοστών; Ποιες ήταν συνολικά οι υψηλές τεχνολογικές γνώσεις του Ευπαλίνου; Αβίαστα βγαίνει το συμπέρασμα, ότι το έργο του Ευπαλίνου πάντα θα έρχεται στο προσκή­ νιο του επιστημονικού και ανθρώπινου ενδιαφέροντος, ως ένα σημείο αναφοράς για τη σύγ­ χρονη επιστήμη. ΕΥΚΛΕΙΔΗΣ Α' 86 τ.2/12


Η

Τάξη

Ιστορία της Αριθμητικής

=======

Γεώρ Ύιος Ωραιόπουλος

2. Κλασματικοί αριθμοί ή κλάσματα

Καλούμε αριθμό κλασματικό ή κλάσμα έναν αριθμό που μετρά ένα ή περισσότερα ίδια μέρη μιας ακεραίας μονάδας του ίδιου μεγέθους. Έτσι αν την ακέραια μονάδα ενός μεγέθους τη χωρίσουμε σε επτά (7 ) ίσα μέρη και πάρουμε το 1 με ίσο μέρος που θα το λέμε Ορισ μός:

«ένα έβδομο» και θα το σημειώνουμε .!. ή τα 3 από τα ίσα μέρη θα έχουμε «τρία έβδομα» ή � . 7 7

Οι αριθμοί .!. � είναι κλάσματα. Ο 7 που είναι κάτω από . τη γραμμή δείχνει το πλήθος από τα 7 7 ,

ίσα μέρη στα οποία κόπηκε η ακέραια μονάδα λέγεται παρονομαστής του κλάσματος � Ο 7 αριθμός 3 που είναι πάνω από τη γραμμή δείχνει από πόσα ίσα μέρη ίσα με την κλασματική .

μονάδα .!. λέγεται αριθμητής του κλάσματος � . 7 7 Ο αριθμητής και ο παρονομαστής ενός κλάσματος καλούνται όροι του κλάσματος. Αν το ευθύγραμμο τμήμα ΑΒ το διαιρέσουμε σε 4 ίσα μέρη το ΑΑ' είναι τα

� 4

του ΑΒ.

Αλλά αν το διαιρέσουμε σε 8 ίσα μέρη το ΑΑ" είναι τα � του ΑΒ. Αλλά ΑΑ'= ΑΑ" δηλ. 8 � = � Αυτά τα δυο κλάσματα καλούνται ισοδύναμα ή ίσα. 4 8 Αν οι όροι κλάσματος πολλαπλασιαστούν ή διαιρεθούν με τον ίδιο αριθμό προκύπτει κλάσμα ισοδύναμο με το αρχικό. Στην περίπτωση της διαίρεσης λέγεται απλοποίηση. Να

70 . Αναλύουμε τους 2 όρους σε γινόμενα πρώτων παραγόντων 72 90 2 · 3 2 5 , 72=23'32 .Μέγιστος κοινός διαιρέτης είναι 2'32 με αυτόν διαιρούμε τους 2 όρους 90 90 : ( 2·32 ) 90 · 18 5 · = - . Το κλάσμα που δεν απλοποιείται λέγεται ανάΎWfΟ. 72 72 : ( 2 . 3 2 ) 72 : 18 4

απλοποιηθεί το κλάσμα =

=

- =

Σύγκριση κλασμάτων

�4

=

ι.! επειδή 5 4 · 1 + 1 . 'Οταν ο αριθμητής είναι μεγαλύτερος από τον παρονομαστή το 4 =

κλάσμα λέγεται καταχρηστικό όπως

30

-

6

=5 .

'Οταν δυο κλάσματα έχουν ίσους παρονομαστές τότε λέγονται ομώνυμα. Αν έχουν και 15 1 1 ' άνισους aριθμητές τοτε >-. 18 18 'Οταν δυο κλάσματα έχουν άνισους παρονομαστές τότε λέγονται ετερώνυμα. Αν έχουν και ΕΥΚΛΕΙΔΗΣ Α' 86 τ.2/13


------

Η Ιστορία της Αριθμητικής

------

, παρονομαστη, 10 10 . , μικροτερο , μεγαλ:υτερο , αυτο, που εχει , αριθμητες, ' ειναι ισους 7 5 49 έχουν ίσα 7"70=10"49 ενώ στα άνισα κλάσματα είναι Δυο ισοδύναμα κλάσματα }_ = 10 70 άνισα τα αντίστοιχα γινόμενα όπως στα κλάσματα �, 8 }_ 10 έχουμε 5 ·1 Ο 7 8 . Αν διαιρέσουμε την ανισότητα αυτή με το γινόμενο των παρονομαστών το σημείο της , , 5 · 10 7 ·8 η, 5 7 διατηρειται ανισοτητας 8 · 10 8·10 8 10 -

<

--

<

--

-

<

-

<-

·

.

Τ ροπή ετερωνύμων κλασμάτων σε ομώνυμα 12 u

9 υ

8 υ

5 υ

12 u

Διαιρούμε το ΕΚΠ των 4 . παρονομαστών με κάθε . . 5 7 8 1 1 5 η, 5 6 7 9 8 8 η, 60 63 64 66 6' 8' 9' 12 ' 6' 72 ' 72 ' 72 ' 72 ' 72 ' 72 ' 72 παρονομαστή. Το πηλίκο το πολλαπλασιάζουμε με τους 2 .Κ.Π. (6, 8, 9, 12)=72 όρους του κλάσματος. Ε Π ράξεις α) Πρόσθεση και αφαίρεση Πρέπει τα κλάσματα να είναι ομώνυμα. Στο πρόβλημα και ομοειδή. 7 + 2 + 1 3 = 7 . 5 + 2 . 4 1 3 . 3 = 35 8 1 9 = 1 52 = 1 13 20 12·5 15·4 + 20 ·3 60 + 60 + 60 60 ϊ5 α) 12 ϊ5 15 1 = 17 33 - 7 31 = 10 32 = 18-7 β) 18-7 3 14 γ) Πολλαπλασιασμός και διαίρεση. Οι μεικτοί γίνονται κλάσματα. 4 ·5 -�= 32 = 3� 4-�-1� = 9 5 9 5 9 9 16 .. 3 2 - 16 .. 3 1 - 16 . 25 - 16 . 8 - 128 25 16 25 8 25 8 25 25 625 Πρόβλη μα. 'Ενα ύφασμα κοστίζει 62_!_€. Αν έπαιρνε 3m ακόμη θα κόστιζε 100€. Πόσα 2 ήταν τα πρώτα m; Σκέψη: Τα 3m θα κοστίζουν τη διαφορά 100-62 _!_. Από την τιμή των 3m θα βρούμε την 2 τιμή του 1m με διαίρεση μερισμού. Από την τιμή του 1m και την τιμή των πρώτων m θα τα βρούμε με διαίρεση μέτρησης. Πράξεις: Τα 3m αξίζουν 100 - 62_!_ = 99� - 12_!_ = 37 _!_.Τα 1m αξίζουν 2 2 2 2 1 3=75 : 3 = 75 :3 = 25 = 12-.Τα 1 πρωτα 1 1 125 25 125 2 5 , θα ειναι , 62-:12-37-: 2 2 2 2 2 25 2 2 2 2 2 Απάντηση: Τα ζητούμενα ήταν 5m. .

=- :

-

=-·- =

Δεκαδικά κλάσματα ή δεκαδικοί αριθμοί

Καλούμε δεκαδικό κλάσμα κάθε κλάσμα με παρονομαστή 1 Ο, 100,1000 ....δηλαδή μια δύναμη του 1 Ο, 1 ον και ν ακέραιος όπως τα 973 = 900 + _!!!_ + _3_ = _2_ + _2._ + _3_ = 0 973 1000 1000 1000 1000 10 100 1000 ' Η τελευταία γραφή του δεκαδικού κλάσματος σε μορφή ακεραίου. Γράφουμε στην αρχή το Ο ΕΥΚΛΕΙΔΗΣ Α' 86 τ.2/14


------

Η Ιστορία της Αριθμητικής

-------­

που σημαίνει ότι ο αριθμός αυτός δεν έχει ακέραιες μονάδες κατόπιν ένα (,) την υποδιαστολή και δεξιά τον αριθμητή του δεκαδικού κλάσματος. Ο αριθμός 0,973 λέγεται δεκαδικός αριθμός. Αν έχουμε ένα καταχρηστικό δεκαδικό κλάσμα ή μεικτό αριθμό εξάγουμε τις ακέραιες 3017 = 3 __!2_ = 3, ο 17 . = 1 .2.. = 1, 7 μονάδες που τις γράφουμε πριν την υποδιαστολή όπως .!2 ' 1000 1000 10 10 Τα ακραία μηδενικά στους αριθμούς δεν τους μεταβάλλουν 0,35=0,350 , 1,08=001,08 Όταν μεταβάλλεται η θέση της υποδιαστολής μεταβάλλεται και ο δεκαδικός. Έστω οι αριθμοί 2,45 , 24,5 , 0,245 . Ο 2°ς είναι 10πλάσιος του πρώτου, ο τρίτος το 1/10 του πρώτου. Έτσι ο πολλαπλασιασμός ή η διαίρεση ενός δεκαδικού αριθμού με 1 Ο, 100, 1000 ισοδυναμεί με μετάθεση της υποδιαστολής του προς τα δεξιά ή αριστερά όσα είναι τα Ο του 1 Ο, 100,1000. Σύγκριση δεκαδικών αριθμών 9,4>7,48 , 0,38<0,4 Πρόσθεση και αφαίρεση δεκαδικών αριθμών γίνεται αν βάλουμε τον ένα κάτω από τον άλλο ώστε οι υποδιαστολές να βρίσκονται στην ίδια στήλη. 3,27 24,0 325,746 49 0,3 14.327 -0.8 39.5 42,77 9.27 340,073 48,2 33,57 Πολλαπλασιασμός: 14,50xl,07000= 14,5x1,07=15, 5{ 5 Διαί ρεση: 37,25:8=3725:800=4,65625.Αυτές οι πράξΕις γίνονται με ηλεκτρονικό υπολογιστή. Τροπή δεκαδικού σε κλάσμα και κλάσματος σε δεκαδικό. 307 36 = .2._ = 0 9 4 = 1 .!. = 1 3333... 3' 07 = 3 2_ = 100 100 ' 40 1 ο ' ' 3 3 ' Αν κλάσμα έχει παρανομαστή με παράγοντες 2 ή 5 τρέπεται σε τερματιζόμενων δεκαδικό. Αν όμως έχει και άλλους παράγοντες τρέπεται σε ατέρμονο δεκαδικό αριθμό. 35 . Τροπή δεκαδικού περιοδικού σε κλάσμα Ο, 353535 = 99 Κάθε απλός δεκαδικός περιοδικός αριθμός ισούται με κλάσμα που έχει αριθμητή την περίοδο και παρονομαστή τόσα 9 όσα ψηφία έχει η περίοδος . Αν δεν αρχίζει η περίοδος μετά την υποδιαστολή όπως ο 0,73932 . . . τον πολλαπλασιάζουμε 39 7 732 99 = 7 ·99 + 39 = = -και τον διαιρούμε με το 1 Ο, 10 ·0,1073339 = 7,3939 10 10 99 ·10 990 73982-73 , Παραδειγμα: Ο, 73982982 = 99000 Ώστε κάθε μεικτός περιοδικός αριθμός ισούται με κλάσμα που έχει αριθμητή ακέραιο που αποτελείται από το μη περιοδικό τμήμα με μια περίοδο πλην το μη περιοδικό μέρος και παρονομαστή ακέραιο με τόσα 9 όσα τα ψηφία της περιόδου, ακολουθούμενα από το Ο όσα τα τμήματα του περιοδικού μέρους. Σύνολο των ρητών αριθμών είναι οι φυσικοί αριθμοί, οι κλασματικοί και οι δεκαδικοί τεpματιζόμενοι και πεpωδικο\ , Q = I = ; όπου α, β ακέραιοι, β Ο . ----

{χ χ

Να τραπούν σε ανάγωγα κλάσματα οι ρητοί αριθμοί: α) 261,25, β) 8 � ·103 , γ) 1,010 δ) 0,5454 . . . , ε) 0,03703L_

*

}

Α 11 .

ΕΥΚΛΕΙΔΗΣ Α' 86 τ.2/15

. .

, στ)

0,89393 ...


Συμμετρία =======Πέτρος Νικολουδάκης και Σπύρος Δη μόπουλος

Αξονική

1.

ΣΥΜΜΕΤΡΙΑ

I

2.

Κεντρική

I

3 . ως προς επίπεδο

1 . ΑΞΟΝΙΚΗ ΣΥΜΜΕΤΡΙΑ

α) Έστω άξονας συμμετρίας η ευθεία (ε). Από ένα σημείο του σχήματος π.χ. το Α φέρνουμε κάθετη στην ευθεία (ε)( με το γνώμονα), και εστω Δ το ίχνος της καθέτου στην (ε), προεκτείνουμε την ΑΔ και στη προέκτασή της ορίζουμε τμήμα ΔΑ'=ΑΔ. Τότε Α ' συμμετρικό του Α ως προς τον άξονα (ε). Συμβολικά : Α 'Συ μμ. ( ε ) Α . Η ευθεία (ε) είναι και η μεσοκάθετη του ΑΑ. Εδώ πρέπει να επισημάνουμε ότι δεν χρειάζεται να βρούμε τα συμμετρικά όλων των σημείων ενός σχήματος, παρά μόνο των χαρακτηριστικών του σημείων. ++

Παραδείγματα σχη μάτων και των συμμετρικών τους ως προς ευθεία ε (αξονική συμμετρία)

}

Για ένα σημείο Α. ΑΑΒ ε' = Α τότε Α ' Συ μμ. ( ε ) Α Β Β 2. Για ένα ευθύγραμμο τμήμα ΒΓ. ΑΚ.ε ΚΑ ' = ΚΑ τότε Α'Συμμ. ( ε )Α τότε και Β Λε ΛΒ ' = ΛΒ τότε Β'Συμμ. ( ε )Β ευθ. τμ.Α 'Β 'Συμμ. ( ε )ΑΒ 1.

++

} }

3.

Β

κ

Λ

ε

Και γενικά για οποιοδήποτε άλλο σχήμα, π.χ. τρίγωνο ΑΒΓ.

} } }

ΑΛε ΛΑ ' = ΑΛ τότε Α 'Συμμ. ( ε )Α ΒΚε ΚΒ ' = ΒΚ τότε Β 'Συμμ. ( ε )Β τότε και Γ Με ΜΓ ' = ΓΜ τότε Γ 'Συμμ. ( ε )Γ τμιγ. Α'Β' Γ 'Συμμ. ( ε )τριγ.ΑΒ Γ . Με δίπλωση του σχήματος πάνω στον άξονα ε, κατά 1 80 διαπιστώνουμε ότι : τρίγ. ΑΒΓ = τριγ . Α 'ΒΤ'

Β'

Γ8ΓιιΜ

�,

ΕΥΚΛΕΙΔΗΣ Α' 86 τ.2/16

Α'


------

Συμμετρία

-------

2. ΚΕΝΤΡΙΚΉ ΣΥΜΜΕΤΡΙΑ

α) Έστω Κ κέντρο συμμετρίας. Από ένα σημείο του σχήματος π.χ. Α φέρνουμε το ευθ. τμήμα ΑΚ και στην προέκταση του ΑΚ προς το Κ, ορίζουμε ευθ. τμήμα ΚΑ '=ΑΚ. Τότε το Α ' είναι συμμετρικό του Α, ως προς κέντρο συμμετρίας το σημείο Κ. Συμβολικά: Α ' Συμμ. ( Κ )Α . Προφανώς το κέντρο συμμετρίας είναι το μέσο του ΑΑ', Εδώ πάλι πρέπει να πούμε ότι δεν χρειάζεται να βρούμε τα συμμετρικά όλων των σημείων ενός σχήματος, για να έχουμε το συμμετρικό του σχήμα. Βρίσκουμε τα συμμετρικά των χαρακτηριστικών το σημείων. Παραδείγματα σχη μάτων και των συμμετρικών τους ως προς κέντρο Κ (κεντρική συμμετρία) 1 . Για ένα σημείο Α. Το Κ είναι το κέντρο συμμετρίας . Αν ΚΑ '=ΑΚ. Τότε : . Α'Συμμ. ( Κ )Α . 2.

Για ένα σχήμα , π.χ. μια γωνία χόψ . Το Κ είναι το κέντρο συμμετρίας . Το Α είναι τυχαίο σημείο της Οχ και το Β είναι τυχαίο σημείο της Οψ

ΚΟ' = ΟΚ τότε : Ο'Συμμ. ( Κ )Ο Αν ΚΑ' = ΑΚ τότε : Α'Συμμ. ( Κ )Α τότε και : χ ' Ο'ψ ' Συμμ. ( Κ ) χόψ . ΚΒ ' = ΒΚ τότε : Β 'Συμμ. ( Κ )Β Παρατηρούμε ότι οι δύο γωνίες χΟψ και χΌ 'ψ ' είναι ίσες, επειδή η μία συμπίπτει με την άλλη, αν την περιστρέψουμε γύρω από το Κ κατά 1 80σ. 3. ΣΥΜΜΕΤΡΙΑ ΩΣ ΠΡΟΣ ΕΠΙΠΕΔΟ

Στην Α ' Γυμνασίου δεν εξετάζεται η συμμετρία αυτή και νομίζουμε πουθενά στην Μέση κπαίδευση. Για να έχουμε συμμετρία ως προς επίπεδο, βασικά χρειάζεται ένα επίπεδο Ε (επίπεδο συμμετρίας). Π.χ. εάν ένας καθρέπτης είναι το επίπεδο συμμετρίας, τότε κάθε αντικείμενο με το είδωλό του στον καθρέπτη είναι συμμετρικά.

}

I ΑΚ ( π) και ΚΑ ' = ΑΚ τότε : Α 'Β 'Συμμ. ( π ) ΑΒ , επομένως και Α 'Β ' = ΑΒ. ΒΛ ( π) και ΛΒ' = ΒΛ ·

ΓΕΝΙΚΕΣ ΠΑΡΑΤΗΡΗΣΣΕΙΣ ΣΤΙΣ ΣΥΜΜΕΤΡΙΕΣ(ΑΞΟΝΙΚΗ-ΚΕΝΤΡΙΚΗ) 1 . Μια ευθεία ή ένα σημείο λέγεται άξονας συμμετρίας ή κέντρο συμμετρίας ενός σχήματος (Σ), εάν το συμμετρικό (Σ ') του σχήματος ως προς ευθεία ή ως προς κέντρο, είναι το ίδιο το σχήμα. Έτσι. ο κύκλος έχει άπειρους άξονες συμμετρίας, είναι οι ευθείες των διαμέτρων του και ένα κέντρο συμμετρίας, το κέντρο του. 2. Σε κάθε συμμετρία τα σχήματα είναι ίσα μεταξύ τους. ΕΥΚΛΕΙΔΗΣ Α' 86 τ.2/17


------

Συμμετρία

-------

3. Για να βρούμε το συμμετρικό ενός σχήματος σε κάθε συμμετρία, βρίσκουμε τα συμμετρικά των χαρακτηριστικών του σημείων. Π.χ. για ένα τετράγωνο βρίσκουμε τα συμμετρικά των κορυφών του, για ένα κύκλο το συμμετρικό του κέντρου του, κ.λ.π. 4. Πολλά σχήματα έχουν και άξονα συμμετρίας και κέντρο συμμετρίας. Π.χ. το τετράγωνο έχει 4 άξονες συμμετρίας και ένα κέντρο συμμετρίας, που είναι το σημείο τομής των αξόνων συμμετρίας του.

Η συμμετρία

ΣΥΜΜΕΤΡΙΑ - ΑΡΜΟΝΙΑ - ΑΝΑΛ ΟΓΙΑ - ΙΣΟΡΡΟΠΙΑ

κρατά την ισότητα των σχημάτων, η Α ρμονία - Α ναλογία κρατούν την ανισότητα και την ομοιότητα, η δε Ισορροπία κρατά τη διαφορετικότητα. Όλα μαζί την απόλυτη ομορφιά του σύ μπαντος. I

ΑΣΚΗΣΗ ΓΙΑ ΛΥΣΗ Α12• Χαράξτε ένα τετράγωνο ΑΒΓΔ με πλευρά 3cm. Να φέρετε τη διαγώνιό του ΑΓ και μια ευθεία ε που να διλερχεται απο την κορφή Α. Να σχεδιάσετε ι)το συμμετρικό Α'Β'Γ' Δ' του τετραγώνου ΑΒΓΔ με άξονα συμμετρίας την ευθεία ε και ιι) το συμμετρικό του τετραγώνου ΑΒΓΔ με κέντρο συμμετρίας την κορυφή του Α. α) Τι παρατηρείτε γtα τα συμμετρικά του τετραγώνου ΑΒΓ Δ; β) Τι σχήμα είναι το ΔΒΒ 'Δ' ; και γιατί; Ποιοι είναι οι άξονες συμμετρίας του ΔΒΒ 'Δ ' ; γ) Να εξετάσετε εάν το εξάγωνο ΔΓΒΒ'Γ' Δ' έχει άξονες συμμετρίας και να δικαιολογήσετε την άποψή σας. δ) Το σημείο Α για ποια σχήματα είναι κέντρο συμμετρίας; ε) Πόσα ορθογώνια τρίγωνα υπάρχουν στο σχήμα και ποια είναι ίσα μεταξύ τους; στ) Πόσα τραπέζια έχει το σχήμα, ποια είναι συμμετρικά ως προς άξονα ή κέντρο; Υπάρχουν παραλληλόγραμμα στο σχήμα, αν ναι ποια είναι; J

ΕΥΚΛΕΙΔΗΣ Α' 86 τ.2/18


Σχέδ ιο μαθήματος στο Πυθαγόρε ιο Θεώρημα της Β 'Γυμνασίου Τάξη

======

Ιωά�ς Ν. Φόβος

Μαθηματικό περιεχόμενο-διάρκεια

Μ' αυτό το σχέδιο μαθήματος ο καθηγητής, μετά από την έννοια του εμβαδού, που έχει εισαχθεί στα

αμέσως προηγούμενα μαθήματα, θα εισάγει το Πυθαγόρειο θεώρημα, δίνοντας την ευκαιρία στους/ις μαθητές/ριες να γνωρίσουν τη στενή συσχέτιση της Άλγεβρας με τη Γεωμετρία. Επειδή η ιστορική του διάσταση είναι πολύ ενδιαφέρουσα, θα χρησιμοποιηθούν δραστηριότητες από την ιστορία των μαθηματικών, όπως άλλωστε αναφέρεται στο Α.Π.Σ. και στο βιβλίο του εκπαιδευτικού. Ο χρόνος εκτιμάται σε 3 διδακτικές ώρες. Διδακτικοί στόχοι Οι μαθητές/ριες επιδιώκεται να: 1 . γνωρίζουν το Πυθαγόρειο Θεώρημα (Π. Θ.) και το aντίστροφό του,

2. γνωρίζουν μερικές από τις αποδείξεις του, 3. γνωρίζουν την ιστορία του Πυθαγόρα και τη χρήση του θεωρήματος από διάφορους λαούς, 4. ελέγχουν αν ένα τρίγωνο με γνωστές πλευρές είναι ορθογώνιο, 5. γνωρίζουν τις Πυθαγόρειες τριάδες. �

Μέσα διδασκαλίας-υλικά γεωπίνακας, χαρτόνια, γεωμετρικά όργανα και υπολογιστές χειρός, φύλλα εργασίας, πίνακας διδασκαλίας και βιντεοπροβολέας.

Περιγραφή-δομή του μαθήματος-δραστηριότητες Γίνεται μια σύντομη επανάληψη των εννοιών του ορθογωνίου τριγώνου και των στοιχείων του, των ιδιοτήτων των γωνιών και των πλευρών του, καθώς και των τύπων των εμβαδών του τετραγώνου και του ορθογωνίου τριγώνου. Στη συνέχεια χωρίζονται οι μαθητές/ριες σε ομάδf:ς των τεσσάρων ατόμων και τους δίνεται ένα φύλλο εργασίας και χαρτόνια και τους ζητείται να συμπληρώσουν τα πάζλ, όπως στα παρακάτω σχήματα, με τη βοήθεια του τριγώνου του σχ. l και να συγκρίνουν το εμβαδόν του τετραγώνου που απομένει στο σχ.2 μα το άθροισμα των εμβαδών των τετραγώνων που απομένουν στο σχ.3, γράφοντας την αλγεβρική σχέση. Δραστηριότητα 1

ι, Σχήμα 1

Σχήμα 2

Σχήμα 3

Για να γίνει η γεωμετρική «ανακάλυψη» και κατανόηση του Π.Θ. δίνονται κι άλλες δραστηριότητες, όπως οι παρακάτω σε μορφή πάζλ και ζητείται μετά από τα παιδιά να γράψουν και να πουν τη σχέση που φαίνεται να προκύπτει. ΕΥΚΛΕΙΔΗΣ Α' 86 τ.2/19


----

Σχέδιο μαθήματος στο ΠυθαΎόρειο Θεώρημα της Β' Γυμνασίου

-----­

Δραστηριότητα 2

Το τετράγωνο που σχηματiζεται με πλει>ρά την uποτεί­ νοuσα ενός ορθοyωνίοu τριyιόνου το χωρiσαμε σε πέντε σχήματα : τtσσερα ορθοyώνια τρίyωνα και ένα τετράyωνο. Μπορείτε με mnά τα πέντε σχήματα να σχηματίσετε δuο τετράyωνα που να έχουν πλεvρά, καθεμιά από τις κά­ I I θετες πλεvρές του ορθοyωνίοu τριyώνοu;

; -� --

�--�--: I

I_L

"' --

.

I I

J

_ _

J

------ -�--

-- __ ___

_

_j

_

(Μπορείτε αν νομίζετε ότι θα σας διevκολ6vει v� σχε­ διάσετε το σχήμα σ 'έvα χαρτί και με ένα ψαλίδι vα κόψετε τα πέντε σχήματα.

Ποια σχέση σuνδέει τα εμβαδά των τρώw τετραγώνων. Δραστηριότητα 3 Στο διπλανό σχήμα. 1. Να υπολοyίσετε το εμβαδόν των τετραyώνων Εικαι Ez ποu έχοuν πλεvρές τις δυο πλευρές του ορθοyωνίοu τριyώνου με μονάδα μέτρησης το τριγωνικό πλακώα που είναι στην άκρη yραμμοσιαασμtνο. 1. Με την ίδια μονάδα μέτρησης να υπο).σγίσετε το εμ­ βαδόν Ε του τετρcryιόνου που έχει πλευρά την υποτεί­ νουσα του ορθοyωνίοu τριyώνου 3. Ποια σχέση συνδέει τα εμβαδά των τριών τετραyώνων; Δραστη ριότητα 4

Σ' ένα χαρτί μιλιμετρέ είναι σχεδιασμtνο ένα σχέδιο ί­ διο με το προηyο6μενο. 1. χω.,ίστε καθένα από τα τετράyωνα που txouν πλευρά τηv κάθετη πλευρά του ορθοyωνίοu τριyώνου σε δοο τρiyωνα, φέρνοντας μια διαyώνιο του. 1. Τοποθετήστε τα παραπάνω τρiyωνα πάνω στο τετ� yωνο με πλευρά την υποτείνουσα του ορθοyωνίοu τρι.­ yώνου. 3. Ποια σχέση σuνδέει τα εμβαδά των τριών τετραyώνων; Δραστηριότητα 5

ι·

t

Στο btnM.vό σ.ιrιiμιι .. ποvμι

t"ιι oeOoyιiιvto

• etyι..ι "o

(Α•,Ο')

AJT

μι μJiΚοΙ vnοtι\,οuσιιι ιι Κ41 μJiKI'I κ.ιΟι­ tι•ι" n:A.ιveriι" •

κιι1 y rrr.ιtιetκ.ι

τοu

τetyιiι,ou

.,ποuμι

κιιτιισκ&u&σ&1

' " e.ιyι..ι,ιι μι μJiKI'I n:A.ιueιiιv a, στοtπιι Xel'lσtμonotιiι"t&! τ ιι

t

• e'ιι

κιι1 y ιιvτtπer.ιιιιιηστ.ι

«Κομμ.ί.ηιι,. nov ιιnοτ ι:Α.ο1Ιv τιι τιτ e-ιvωvιι "''" ι.:46ιτr.ι" nlιueιiιv, ιιnοeιtτι vιι .. γ&μtσ&τι,. το J<&y.t:A.o vκet'lo τ ι τ e-ιvωvο tl'l! unοτ ιtvοuσιιι '+"-ι>Ι<ό'Ιοηιιι ιιι<et•ιiιΙ τιι πer.ιιιιιηστ.ι ιcοιιιώηιι πr.ιeΙΙ το &vιι vιι ιntκ&:A..!ntιt το .ιllo,

ΕΥΚΛΕΙΔΗΣ Α' 86 τ.2/20


----

Σχέδιο μαθήματος στο Πυθαγόρειο Θεώρημα της Β' Γυ μνασί()υ Δραστηριότητ α 6

γ

β γ

β

Δfνονται οκτώ /σα ορθογώνια τρfγωνα με κtιθετες πλευρtς β, γ και υποτεfνουσα α και τρfα τετράγωνα με πλευρtς α, β, γ αντlστοιχα.

� γ

[Ξ] β

α) Να υπολογfσετε τα εμβαδά ε, Ε, Ε1, Ε2 των διπλανών τριγώνων και τετραγώνων.

α

/3) Να τοποθετήσετε κατάλληλα τα τρfγωνα και τετράγωνα, ώστε να σχηματlσουν δύο νta τετράγωνα, πλευράς (β + γ).

α

Μετά από κάθε δραστηριότητα ακολουθεί μια συζήτηση για τη 't σχέση που προκύπτει και μετά την � :, ....... τελευταία ο δάσκαλος αναφέρει ότι η σχέση αυτή είναι από τα πιο των θεωρήματα διάσημα Πυθαγόρειο το μαθηματικών, θεώρημα. Εκείνη τη στιγμή κάνει μια ιστορική αναδρομή, αναφέρεται στη ζωή, το έργο και τη θεωρία του Πυθαγόρα, καθώς και στη χρήση του Αιγύπτιους, τους από Π.Θ. Βαβυλώνιους, Κινέζους και Ινδούς, ενώ λέει επίσης ότι η δραστηριότητα 5 αποδείχθηκε από το Σωκράτη (500 π.Χ.), όπως λέει ο Πλάτωνας στο διάλογο του «Μένων)) και η Εικόνα 1 δραστηριότητα 3 από τους Κινέζους με την τεχνική του πάζλ, που υπάρχει στο βιβλίο του Τσου-Πέι. Σ'αυτό το σημείο προβάλλει με το βιντεοπροβολέα την εικόνα με ένα ελληνικό κείμενο του θεωρήματος καθώς και πέντε μεταφράσεις σε διάφορες γλώσσες και το διατυπώνει στην αρχαία γλώσσα, όπως σώθηκε στα Στοιχεία του Ευκλείδη. (Εικ. l). Στη συνέχεια τους(ις) ζητά να βρουν τη σχέση των αριθμών του παρακάτω πίνακα 1 του φύλλου εργασίας, που βρέθηκε σε Βαβυλωνιακή πινακίδα Plimton 322, αναφέροντας εδώ κάποια ιστορικά στοιχεία γι' αυτήν και προβάλλοντας τις εικόνες 2 και 3. Αναφέρει τις Πυθαγόρειες τριάδες, την Πλατωνική και τη Πυθαγορική μέθοδο που δίνει τέτοιες τριάδες αριθμών και ζητά να κάνουν τη δραστηριότητα 7 του φύλλου εργασίας. (..ί .. �"""/ "' ":"

Εικόνα 2

β . � 1 19

σ

j

120

-----

3,456 .·

,

;

γ ' .. -- --r-- - ---- . �Ι-----ι 1β!_.. , .ι.. �·

·---

·-

·· - - -·

3,367

. .•

!

+ �-�--�......... 13.000 � 12,700 j 4.ιισο

..

. - - - -�--

- --·

72

ι

t !

4 so1

· - -- «·

β5

JI

·

7eΜι

�.

--�--

18,541 97

ΕΥΚΛΕΙΔΗΣ Α' 86 τ.2/21

Πίνακας 1


------

Σχέδιο μαθή ματος στο Πυθαγόρειο Θεώρη μα της Β' Γυμνασίου

------­

Δ ρ αστη ρ ιότητα 7 Οι Βαβυλώνιοι γνώριζαν το θεώρημα αυτό 1 000 χρόνια νωρίτερα, την δόξα όμως την πήρε ο Πυθαγόρας που πρόr τος το απέδειξε. Σε μια mνακίδα της πρώτης βαβυλωνια­ κής περιόδου( 1900-1 600 π.Χ) βρίσκουμε Ύραμμένο το ε­ ξής πρόβλημα : «Ένα δοκάρι που έχει μήκος 30 (ακουμπά ιcατακόρυφα σε έναν τοίχο). Το πάνω άκρο του 'fλιστpάει προς τα κά­

τω κατά 6. Πόσο απομακρύνεται το κάτω άκρο;» Μποοε(τε να το υπολοviσετε:

Μετά την ολοκλήρωση της δραστηριότητας, κάθε ομάδα παίρνει από ένα γεωπίνακα με τα λαστιχάκια του και ένα φύλλο εργασίας με σχήματα της μορφής : •

:.;.1 ι

Δ ι

• • •

όπου ζητείται να κατασκευαστούν τα αντίστοιχα ορθογώνια τρίγωνα. Στη συνέχεια δίνεται η δραστηριότητα 8, με σκοπό την επαλήθευση του aντίστροφου του Π.Θ. και τον έλεγχο αν ένα τρίγωνο με γνωστές τις πλευρές του είναι ορθογώνιο. Δ ρ αστη ρ ιότητα 8 στην αρχαία Αi.,υπτο οι αρπεδονάπτες σημάδευαν πάνω στην -yη ορθές γωνfες. ΧρησιμοποιοΟΟαν ένα σχοινί με 13 κόμποuς, την αρπε­ δόνη όπως την ονόμαζαν, οι οποiοι απείχαν μετο.ξU τοuς ίσες αποστάσεις. Την χώριζαν δε σε τρία κομμάτια ταιν 3, 4 και 5 μονάδων. Με δυο καρφιά στερέωναν στη -yη τεντώνο­ ντας το μεσα{ο κομμάτι ταιν 3 μονάδων ιcαι ένωναν τεντώ­ νοντας τοuς δοο ακρινοός tcόμποuς σχηματiζοντας μια ορθή Ύαινία. Πό>ς είμαστε οtyουροι ότι σχηματi.σθηκε ορθή yαινiα; Τι λέτε στην αρχαία ΑίΎUΠtΟ ήξεραν να χρησψ.οποιοον το Πuθαyόρειο θεό>ρημα ;

Μ

·'

Σ'αυτό το σημείο μπορεί να αναφερθεί το Κινέζικο «water weed problem» και το Ινδικό «lotus problem)) δίνοντας έτσι πολλές προοπτικές ενός προβλήματος και μεθόδους επίλυσης από διάφορους πολιτισμούς. Κατά τη διάρκεια διδασκαλίας όλης αυτής της ενότητας δίνονται εργασίες για το σπίτι, όπως αναζήτηση πληροφοριών για τη ζωή και το έργο του Πυθαγόρα, η εύρεση στοιχείων για τις προσπάθειες των Αιγυπτίων, άλλες αποδείξεις του θεωρήματος και ασκήσεις και προβλήματα του σχολικού βιβλίου. Σ'αυτό το σχέδω μαθήματος γίνεται παροχή άμεσων ιστορικών πληροφοριών με μια ωτλή αφήγηση ιστορικών δεδομένων και έμφαση στις ιστορικές πηγές. Η διδακτική προσέγγιση είναι εμπνευσμένη από την ιστορία με φύλλα εργασίας, που περιέχουν ιστορικά προβλήματα. Γίνεται επίσης χρήση χειραπτικών υλικών, του διαδικτύου και άλλου έντυπου υλικού στην αναζήτηση πληροφοριών για τις εργασίες στο σπίτι. Βιβλιογραφία Ευκλείδης Α '. Μαθηματικό περιοδικό για το Γυμνάσιο. Μαθηματικά Β' Γυμνασίου. ΟΕΔΒ. Αθήνα 2007 . Μαθηματικά Β' Γυμνασίου. «Βιβλίο Εκπαιδευτικού)). ΟΕΔΒ. Αθήνα 2007 . ΕΥΚΛΕΙΔΗΣ Α' 86 τ.2/22


ΤΡΙΓΩΝΟΜΕΤΡΙΑ =======

Πέτρος Νικολουδάκης και Σπύρος Δημόπουλος

Ένα από τα πιο δύσκολα προβλήματα στην εκμάθηση των Μαθηματικών στη σχολική διαδικασία είναι η απομνημόνευση των τριγωνομετρικών αριθμών των χαρακτηριστικών γωνιών 0° 30° 45° 60° 90° . Ας * χρησιμοποιήσουμε την «αγχίνου μνήμη»* Τους μηχανισμούς δεν τους ξεχνάμε. '

'

'

'

Μνήμη αγχίνους : το μνημονικό τέχνασμα της ανάκλησης στη μνήμη ενός στοιχείου ( αριθμού , ονόματος κ.λ.π.) με τη βοήθεια άλλου ευκολότερα ανακλητού , με το οποίο έχει κοινά εξωτερικά στοιχεία , λ.χ. για να θυμάται κανείς την χρονολογία της Γαλλικής Επανάστασης ( 1 789) , θυμάται την ακολουθία 7, 8, 9.

Γράψτε τους αριθμούς Βγάλτε τις τετραγωνικές ρίζες τους Δηλαδή ίσο με Διαιρέστε με το 2 Δηλαδή ίσο με

ο JO ο ο 2

1 Jϊ 1 -1 2 -1 2

ο

2 fi fi J2 2 J2 2

3 J3 J3 J3 2 J3 2

4 J4 2 2 2 1

Έτσι έχετε τους τριγωνομετρικούς αριθμούς των χαρακτηριστικών γωνιών ω (0°, 30°, 45°, 60 , 90 )για το ημίτονο ω (ηιιω). Δηλαδή : 60° σο 45° 30° ω 90° 1 J2 J3 -1 ο ημω 2 2 2 Τα πράγματα τώρα είναι πιο απλά για το συνημίτονο αυτών των γωνιών, αφού εργαζόμαστε ' με τον ι'διο τροπο αλλα' με «αναπο δ>ψ> σειρα: 2 4 1 ο 3 Γράφουμε Βγάζουμε τις τετραγωνικές ρίζες τ�υς J4 fi J3 Jϊ JO 1 2 ο Δηλαδή ίσο με J3 fi 2 Διαιρούμε με το 2 ο fi J3 -1 ο

ο 1

'

'

2

Δηλαδή ίσο με

2 J3 2

1

2 fi 2

Οπότε : ω 1 J2 J3 1 συνω 2 2 2 Για την εφαπτομένη έχουμε τη σχέση: εφω = ημωω επομένως συν ω εφω 2 1 Q=O 1

J2

_ _ =

2

2

ο

ο

Δεν ορίζεται

2

Έτσι οι τριγωνομετρικοί αριθμοί των χαρακτηριστικών γωνιών γράφονται συνοπτικά σ' ένα πίνακα: ΕΥΚΛΕΙΔΗΣ Α' 86 τ.2/23

2

2 -1 2

0°, 30°, 45°, 60°, 90°,


------

ω ημω

σο

συνω

1

30° 1 2 -

ο

J3

-

2

J3

Τριγωνομετρία ------

45°

60° J3

-

.fi

-

2

.fi

2 1 2

-

-

2

J3

90° 1

ο δεν ορίζεται

Ι ο εφω 2 Εάν είναι γνωστή μια γωνία σ ' ένα τρίγωνο μέρος κάποιου σχήματος γενικότερα, τότε : 1 ) Εάν χρησιμοποιήσουμε το ημίτονο της γωνίας θα έχουμε τις παρακάτω περιπτώσεις: Ι ) η μω = ΑΓ , άρα η μω = Ά_, άρα β = α · η μω ΒΓ α β 11) ημω = ΑΓ , άρα ημω = Ά. , άρα α = -ημω ΒΓ α 2) Εάν χρησιμοποιήσουμε το συνημίτονο της γωνίας θα έχουμε τις παρακάτω περιπτώσεις: γ αρα , γ = α · συνω , ΑΒ αρασυνω Ι) συνω = -, = -, ΒΓ α α 11) συνω = ΑΒ , άρα συνω = 1.. , άρα γ = __ ΒΓ α συνω 3) Εάν χρησιμοποιήσουμε την εφαπτομένη της γωνίας θα έχουμε τις παρακάτω περιπτώσεις: β 11) εφω = ΑΓ , άρα εφω = � , άρα γ = _L Ι) εφω = ΑΓ , άρα εφω = , άρα β = γ · εφω, γ εφω γ -

ιr

ιιι..

w

ΙΒΙ

ΑΒ

ΑΒ

ΠΑΡΆΔΕΙΓΜΑ 1 ο

Δίνεται τρίγωνο ΑΒΓ με ΑΒ=5 cm, Γ = 25° και φέρνουμε το ύψος ΑΔ και έστω ΔΒ=1,7 cm, τότε

α) Να υπολογίσετε τις γωνίες Β και Γ . β) Να υπολογίσετε τις πλευρές AF και ΒΓ. γ) Να υπολογίσετε το ύψος ΑΔ του τριγ. ΑΒΓ. δ) Να υπολογίσετε το εμβαδόν του τριγ. ΑΒΓ. Λύση ΣΗΜΑΝΤΙΚΗ ΠΑΡΑΤΗΡΗΣΗ

για οποιοδήποτε υπολογισμό γωνιών πρέπει να αναφερθούμε σε κάποιο

ορθογώνιο τρίγωνο. α) Υπολογισμός γωνιών. Από ορθογώνιο τριγ. ΑΒΔ {Δ = 90°) , άρα σJ\.Β=: (μχ,σιΜ3=� =Q34 .

(από τριγωνομ. Πίνακες) Β 70° . Οπότε Α = 180° -{Β + t), άρα Α = 1 80° - ( 70 + 25 ) , άρα Α = 1 80° -95° 85° . β) Υπολογισμός ύψους ΑΔ. Από ορθογώνιο τριγ. ΑΒΔ (Δ 90° ) , άρα η μΒ = � , άρα ημ70° = ΑΔ 5 , άρα ΑΔ = 5 ·ημ70° , άρα ΑΔ = 5 ·0,9397, άρα AΔ :: 4,7cm. ::

=

=

ΕΥΚΛΕΙΔΗΣ Α' 86 τ.2/24


------

Τριγωνομετρία

-------­

γ) Υπολογισμός των πλευρών ΑΓ και ΒΓ. Από ορθογώνιο τριγ. ΑΓΔ {Δ= 90°) , άρα εφΓ = � , ΑΔ , άρα ΔΓ = 4• 7 , άρα ΔΓ = 4• 7 , άρα ΔΓ 10,1 cm. άρα ΔΓ = εφ Γ εφ25° Ο, 4663 Άρα ΒΓ=ΒΔ+ΔΓ= 1,7+10,1=1 1,8cm. Από ορθογώνιο τριγ. ΑΓΔ {Δ = 90° ) ΑΔ = 4• 7 = 4• 7 = 1 1, 1cm η μΓ = ΑΔ , άρα ΑΓ = ημΓ ημ25° 0,4226 ΑΓ δ) Υπολογισμός του εμβαδού του τριγ. ΑΒΓ . Ε ΑΒΓ) = ΒΓ Μ 11• 8 4• 7 = 55 46 cm2 =27, 73cm2 ( � ; ; �

/

ΠΑΡΆΔΕΙΓΜΑ 2 °

Δίνεται ισοσκελές τρίγωνο ΑΒΓ (με ΑΒ=ΑΓ), Α = 44° και BΓ=6cmo Φέρνουμε τα ύψη ΑΔ και ΒΕ ο Να υπολογίσετε 1) τις γωνίες Β και Γ ο 2) τις πλευρές ΑΒ και ΑΓ, 3) τα ύψη ΑΔ και ΒΕ, 4) το εμβαδό του τριγο ΑΒΓ και 5) το λόyο των εμβαδών των τρryώνων ΑΒΕ και ΒΕΓ, δηλ

� �

Λύση

ΑΒ

Ε

ΒΕΓ

Γ

.

Υπολογισμός των γωνιών Β και ο Α. + Β + t = 180°, άρα 44 + Β + Β = 180°, άρα 2Β = 180° -44° = 136°, άρα Β = 68° . 2) Υπολογισμός των πλευρών ΑΒ και ΑΓ. Επειδή το τριγ. ΑΒΓ είναι ισοσκελές, τότε το ύψος ΑΔ είναι και διχοτόμος και διάμεσος, δηλαδή ΒΜ = ΓΑΔ = 22° και ΒΔ = ΔΓ = 3cm. Οπότε από το ορθογώνιο τριγ.ΑΔΓ {Δ = 90°) έχουμε ημΑ1 = : , 3 = 3 = 8cm , επομένως AB=AΓ=8cm. άρα ΑΒ = � = ημΑ 1 ημ22° 0,3746 3) Υπολογισμός των υψών ΑΔ και ΒΕ. Από ορθ . Τριγ. ΑΒΔ {Δ = 90° ) έχομε : εφΒ = ΑΔ ΒΔ , άρα ΑΔ = εφΒ ·ΒΔ, άρα ΑΔ = 3 · εφ68° = 3 · 2,4751 = 7,4cm . Από ορθ. Τριγ. ΒΕΓ ( Ε = 90°) έχουμε ημΓ= =�· άρα ΒΕ=ΒΓ·ημΓ=6·ημ68° =6·0,9272=5,6cm 4) Υπολογισμός εμβαδού τριγώνου ΑΒΓ. Ε ( ΑΒΓ) = ΒΓ · ΑΔ = 6 ' 7• 4 = 44, 4 cm2 = 22,2cm2 2 2 2 5) Υπολογισμός του λόγο των εμβαδών των τριγώνων ΑΒΕ και ΒΕΓ. 1)

ο

1 ( ΑΒΕ) -2 ΑΕ · ΒΕ = = ΑΕ = � =: 2,6· (ΒΕΓ) _!_ ΕΓ · ΒΕ ΕΓ 2,2 2

--

Υπολογισμός ΑΕ και ΕΓ. Από ορθ . Τριγ. ΑΒΕ ( Ε = 90° )

έχουμε : συνΑ = ΑΕ , άρα ΑΕ = συν · ΑΒ = συν44° · 8 = Ο, 7193 ·8 = 5,8cm . ΑΒ Οπότε: ΕΓ=ΑΓ-ΑΕ, άρα ΕΓ=8-5,8, άρα ΕΓ=2,2 cm. ΕΥΚΛΕΙΔΗΣ Α' 86 τ.2/25


Μαθηματ ι κές Σχέσε ι ς ======

ΙΣΟΤΗΤΑΣ ΕΞΙΣΩΣΕΙΣ

Επαληθεύονται με ορισμένες τιμές των γραμμάτων που περιέχουν

Πέτρος Νικολουδάκης και Σπύρος Δημόπουλος

ΑΝΙΣΟΤΗΤΑΣ ΤΑΥΤΟΤΗΤΕΣ

ΑΝΙΣΟΙΣΟΤΗΤΑΣ

Επαληθεύονται με οποιεσδήποτε τιμές των γραμμάτων που περιέχουν π.χ. (α+β)2=α2+2αβ+β2 I) ΕΞΙΣΩΣΕΙΣ ΠΡΩΤΟΥ ΒΑΘΜΟΥ Μ' ΕΝΑΝ ΆΓΝΩΣΤΟ ΓΕΝΙΚΗ ΜΟΡΦΗ αχ+β=Ο ( 1 )

Γενική Λύση - Διερεύνηση

Εάν α:jfO και β οποιοσδήποτε αριθμός τότε η (1) έχει μοναδική λύση, γιατί: αχ+β=Ο ή αχ=-β ή χ = -� α Εάν α=Ο και β =f:. Ο τότε η ( 1) είναι ΑΔΥΝΑΤΗ(δεν επαληθεύεται με κανένα αριθμό), γιατί: αχ+β=Ο ή Οχ=-β ή 0=-β ή β=Ο, άτοπο. 2) Εάν α=Ο και β=Ο τότε η (1) είναι ΤΑΥΤΟΤΗΤΑ(επαληθεύεται με όλους τους αριθμούς), γιατί: αχ+β=Ο ή Οχ=Ο για κάθε χ. Δηλαδή η εξίσωση αχ+β=Ο θα έχει μία λύση ή καμία ή άπειρες, εξαρτάται από τις τιμές των συντελεστών της α, β. Παραδείγματα Δίνεται η εξίσωση .fiκx + 3 = J)μ-2χ (2). Για ποιες τιμές των παραμέτρων κ, μ έχει λύση, είναι αδύνατη ή είναι αόριστη. (ΠΑΡΑΜΕΤΡΟΣ είναι ένα γράμμα ή περισσότερα που εμφανίζονται σε μια εξίσωση και αλλάζουν τη λύση της, ανάλογα με τις τιμές που παίρνουν. Όσο πιο πολλές παραμέτρους έχει ένα πρόβλημα τόσο πιο δύσκολο είναι. Έτσι είναι και στη ζωή μας). Από την (2) έχουμε .J2κχ+3=J3μ-2χ ή .J2κχ+3χ =.J3μ-3 ή (.J2κ+2)χ = .J3μ-3 (ΚΑΝΟΝΙΚΗ ΜΟΡΦΗ), τότε: 2 . .J2 ι;; ι;; 1) Εάν ( ν2κ+ ι;; 2 ) :;t: O ή ν2κ :;t: -2 ή κ :;t: - 2 ή κ :;t: .Ji2· .J2 . .Ji ή κ :;t: - -- ή κ :;t: -ν2 , 2 .J2 , λ':υση, την χ = .J3μ-3 . μ= οποιοσδηποτε. ' ' εχει η εξισωση ν2ι;; +2 2) Εάν .J2κ + 2 =Ο και J)μ-3 :;t: Ο ή .J2κ = -2 και J)μ :;t: 3 ή κ = - .1 και μ :;t: .1 ή 1)

κ = -.fi και μ :;t: J3 , είναι ΑΔΥΝΑΤΗ. (Γιατί Ο · χ = J3 · μ - 3 , με J3 · μ -3 :;t: Ο . 3) Εάν .fiκ + 2 = Ο και J)μ-3 = Ο ή .fiκ = -2 και J)μ = 3 ή κ = - .1 και μ = .1 ή κ = -.fi και μ = J3 , είναι ΑΟΡΙΣΤΗ ή ΤΑΥΤΟΤΗΤΑ, γιατί Ο · χ = Ο για οποιοδήποτε χ. Π) ΕΞΙΣΩΣΕΙΣ ΔΕΥfΕΡΟΥ ΒΑΘΜΟΥ Μ' ΕΝΑΝ ΑΓΝΩΣ'ΓΟ ΓΕΝΙΚΗ ΜΟΡΦΗ �+β +y=Ο (1 ) α;f:Ο (γιατί;) χ

Γεν ική Λύση - Διερεύνηση

Δ = β2 - 4αγ (Διακρίνουσα) ± 1 ) Ε�ν η διακρίνουσα Δ = β2 - 4αγ > Ο τότε η (1) έχει δύο λύσεις ΑΝΙΣΕΣ χ 1 = -β JΔ 2 2α ΕΥΚΛΕΙΔΗΣ Α' 86 τ. 2/26


Μαθηματικές Σχέσεις ------

------

2) 3)

Εάν Δ = Ο τότε η ( 1) έχει δύο λύσεις ΙΣΕΣ. Εάν Δ < Ο τότε η (1) δεν έt,ει λύση πραγματικούς αριθμούς.

Δηλαδή η εξίσωση αχ +βχ+y=Ο (α :f:. Ο) ή θα έχει δύο λύσεις διακεκριμένες ή θα έχει δύο λύσεις ίσες ή καμία λύση,

εξαρτάται από τη διακρίνουσά της Δ. Για να λυθεί μια εξίσωση 2ου βαθμού με ένα άγνωστο, πρέπει να τη φέρουμε στην κανονική μορφή αχ2+βχ+γ=Ο. Τότε υπολογίζουμε την Διακρίνουσα Δ = β2 - 4αγ και αν Δ �Ο πρ�χωράμε σtις ρίζες της με τους τύπους χ 1 - β 2±α.JΔ . ·

2

=

Ε ιδ η<:ές μορφές

Εάν α :f:. Ο, β :f:. Ο και γ = Ο, τότε η (1) γίνεται αχ2+βχ = Ο ή χ(αχ+β) = Ο ή (χ=Ο ή αχ+β=Ο) ή (χ=Ο ή χ = _Ι) α 2) Εάν α :f:. Ο, β = Ο και γ :f:. Ο, τότε η (l) γίνεται αχ2+γ = Ο ή αχ2 = - ή χ 2 γ = - : ή χ = .± Η (προφανώς έχει λύσεις αν γ , α είναι ετερόσημοι) 3) Εάν α :f:. Ο, β = Ο και γ = Ο, τότε η (l) γίνεται αχ2 = Ο ή χ2 = 0 ή χ1 = χ = Ο. 2 η

Π αραδείγματα

)

1 ) Να λυθεί η εξίσωση: 2χ ( χ - 1 γ - 3 (χ - 2) (χ + 2) = 2 (χ - 1 3 + 1 Ο

Λ.ίJση

2χ ( χ-1)2 -3 ( χ-2)( χ + 2) = 2 ( χ-1)3 +10 ή 2χ{ χ2 -2χ + 1) -3 ( χ-4) =2( χ3 -3χ2 + 3χ-1) + 10 ή 2χ - 4χ2 + 2χ - 3χ2 + 12 = 2χ3 - 6χ2 + 6χ - 2 + 10 ή 2χ3 - 4χ2 + 2χ - 3χ2 + 12 - 2χ3 + 6χ2 - 6χ + 2 - 10 = ο

il 2χ3 - 2χ3 - 4χ2 - 3χ2 + 6χ2 + 2χ - 6χ + 12 + 2 - 10 = ο ή -χ2 - 4χ + 4 = Ο ή χ2 + 4χ - 4 = Ο, ( έχουμε όμως Δ= β2 - 4αγ = 42 - 4 1 ( -4) = 16 + 16 = 32), οπότε -β ± JΔ = -4 ± Jfi = -4 ± 4J2 = = -2 + 2J2 , ( αρρητες ριζες , ) 2 2α 2·1 2

Χι

{Χι

·

·

=

'

χ2 = -2-2J2

·

2:1 Να λυθεί η εξίσωση : 2 (χ - 2) (χ + 1) - 5 (χ + 2γ : 2

ή

(χ - )(χ 2

+

Ι ) - 5 (χ + 2 )

2

=

3

=

3

(χ - .J3) (χ .J3)-

(

+

( x-.J3)( χ + .J3) - 15

15

)

ή

2(χ2 + χ - 2χ - 2) - 5(χ2 + 4χ + 4) = 3 χ2 - ( .J3 )2 - 15 ή 2χ2 + 2χ - 4χ - 4 - 5χ2 - 20χ - 20 = 3χ - 9 - 15 ή 2χ2 - 5χ2 - 3χ2 + 2χ - 4χ - 20χ - 4 - 20 + 9 + 15 = Ο ή -6χ2 - 22χ = Ο 6χ2 + 22χ = Ο ή 2χ(3χ+ 1 1) = Ο ή (2χ = Ο ή 3χ + 1 1 = Ο) ή χ = Ο ή χ =

(

-1: .

3) Να λυθεί η εξίσωση: 2 χ ΛίJση 2

.J2)( χ + .J2) + 3 (χ - .J5)( χ + .J5 ) (χ - .J7) ( χ + .J7)

(χ - $)(χ + $)+ 3 (χ - v'S )( x + v'S ) = (χ - FΊ)(χ + FΊ) ή

2 ( χ 2 - 2 ) + 3 ( χ 2 -5 ) = χ 2 -7

=

ή 2χ2 - 4 + 3χ2 - 15 = χ2 - 7 = 0 ή 2χ2 + 3χ2 - χ2 = 4 + 15 - 7 ή 4χ2 = 12 ή χ2 = 3 ή χ = ±.J3 . ΕΥΚΛΕΙΔΗΣ Α ' 86 τ. 2/27


Ομοιότητα =======

ΑΣΧΕΤΑ ΜΕΤΑΞΥ ΤΟΥΣ

ΣΧΕΣΕΙΣ

Πέτρος Νικολουδάκης και Σπύρος Δημόπουλος

ΠΟΛΥΓΩΝΩΝ ΙΣΑ

ΟΜΟΙΑ

Δύο ή περισσότερα πολύγωνα λέγονται ΟΜΟΙΑ μεταξύ τους, όταν έχουν τις πλευρές τους ανάλογες και τις αντίστοιχες (ομόλογες) γωνίες τους ίσες. Λόγος ομοιότητας δύο ομοίων πολυγώνων. είναι ο θετικός πραγματικός αριθμός, που � ._------�r εκφράζει το λόγο δύο αντιστοίχων ή ομολόγων πλευρών τους. ΑΒΓΔ όμοιο με ΑΉ'ΓΔ ' (συμβολικά ΑΒΓΔ ΑΉ'ΓΔ'), τότε ΒΓ = -ΓΔ = -ΔΑ = Α = Α' Β = Β' Γ = Γ ' Δ = Δ και ΑΑΒ 'Β ' Β'Γ ΓΔ' ΔΆ' = λ . Εάν Ο<λ<l τότε έχουμε σμίκρυνση, εάν λ=l τότε τα σχήματα είναι ίσα και εάν λ>l έχουμε μεγέθυνση. Σ ' ένα χάρτη έχουμε σμίκρυνση και η κλίμακά του είναι πάντα μικρότερη του 1 . Σε μια φωτοτυπία όμως, έχουμε κατά βάση ισότητα, αλλά μπορούμε να έχουμε και σμίκρυνση και μεγέθυνση . Τα διάφορα σχέδια (αρχιτεκτονικό, μηχανολογικό, τοπογραφικό κλπ) περιέχουν σχήματα, (σπίτια, μηχανές, αγρούς κλπ), που είναι όμοια με τα αντίστοιχα πραγματικά αντικείμενα και είναι συνήθως σε σμίκρυνση, εκτός εάν παρουσιάζουν αντικείμενο που είναι πάρα πολύ μικρό (κύτταρο, μόριο κ.α.), οπότε έχουμε σχέδιο σε μεγέθυνση. Στις φωτογραφίες που τραβάμε έχουμε σμίκρυνση γενικά, εκτός και αν φωτογραφίζουμε αντικείμενα πάρα πολύ μικρά, π.χ. μια μέλισσα, οπότε χρειαζόμαστε μεγέθυνση. Όμως τα είδωλα των αντικειμένων που σχηματίζονται στα μάτια μας είναι όμοια με τα αντικείμενα, σε σμίκρυνση. Η όρασή μας λοιπόν έχει σχέση με την ομοιότητα, οπότε η ΟΜΟΙΟΤΗΤΑ είναι κάτι το σπουδαίο και τόσο κοντά μας. Ας ασχοληθούμε με τα πιο απλά όμοια σχήματα στο επίπεδο, που είναι τα τρίγωνα. Υπάρχει ένας απλός κανόνας (θεώρημα) : αν δύο τρίγωνα έχουν δύο γωνίες ίσες είναι όμοια. Δηλαδή: Εάν Α = Α' και Β = Β' τότε και t ΘΕΩΡΗΜΑ (1) ΑΒ ΒΓ = ΓΑ = λ και -' = ΑΉ Β'Γ' ΓΑ ' •

8

Λ

Λ

Λ

Λ

Λ

Λ

Λ

Λ

--

{

--

=Γ'}

--

--

Α'

Β'

Υπάρχουν ακόμα δύο θεωρήματα για την ομοιότητα δύο τριγώνων αλλά θα τα δούμε στο Λύκειο. Πάνω σ ' αυτόν τον απλό κανόνα της ομοιότητας δύο τριγώνων στηρίζονται τα πάντα περί ομοιότητας. Σ ' αυτή την τάξη κρατάμε το Θεώρημα (1). Με τη βοήθεια αυτού έχουμε ότι: ΕΥΚΛΕΙΔΙΙΣ Α' 86 τ.2/28


------ Ομοιότητα

------

ΑΒ = ΒΓ = ΓΑ = ΑΒ + ΒΓ +ΓΑ = λ (ο λόγοc_ των περιμέτρων δύο ομοίων Α'Β' Β'Γ ΓΑ ' Α'Β' + Β 'Γ+ΓΑ ' τριγώνων είναι ίσος με το λόγο ομοιότητας). 2) (���') = λ 2 (ο λσyος των εμβαδών δύο ομοίων τριγώνων είναι ίσος με το τετράγωνο του λόγου ομοιότητας). Τέλος μια σημαντική παρατήρηση είναι η εξής: τα κανονικά πολύγωνα με τον ίδιο αριθμό πλευρών είναι όμοια. π.χ. όλα τα κανονικά πεντάγωνα μεταξύ τους, οι γωνίες τους είναι 108°. 1)

Ασκήσεις στα όμοια τρίγωνα 1°) Δίνεται τρίγωνο ΑΒΓ. Πάνω στην πλευρά ΑΒ αρίζουμε ένα σημείο Δ, τέτοιο ώστε AΔ=6cm και

ΑΔ

=

� ΑΒ . Από το Δ φέρνουμε παράλληλη προς την ΒΓ που τέμνει την

5 πλευρά ΑΓ στο Ε. (Δηλ. ΔΕ/ΙΒΓ). Εάν ΕΓ= 7cm, να υπολογίσετε: α) την πλευρά ΑΕ του τργ. ΑΔΕ και τις πλευρές ΑΒ , ΑΓ του τριγ. ΑΒΓ. β) Μεταξύ ποιων αριθμών βρίσκεται η τιμή των πλευρών ΔΕ και ΒΓ.

γ) Να υπολογιστούν οι λόγοι i) (ΑΑΔΕ) ( ΒΓ ) Λύση .

και ii)

( ΑΔΕ) (ΔΕΓΒ )

Α

ΔΕΙIΒΓ

,ΑΔ=βcm

Γ Β .________________�

3 · ΑΒ ή 3·ΑΒ = 30 ή ΑΒ = l Ocm. Επειδή ΑΔ = �ΑΒ ή 6 =�ΑΒ ή 6 = 5 5 5 ΑΕ = ΑΔ ή ΑΕ = � ή 4 . ΑΕ = 42 ή ΑΕ = 10, 5cm . Συνεπώς ΑΓ=10,5+7= 17,5cm. ΕΓ ΔΒ 7 4 β) Από τριγωνική σχέση στο τριγ. ΑΔΕ είναι: ΑΕ-ΑΔ<ΔΕ<ΑΕ+ΑΔ ή 10,5 - 6<ΔΕ<10,5+6 ή 4,5cm<ΔE<16,5cm , ομοίως στο τριγ. ΑΒΓ είναι ΑΓ-ΑΒ<ΒΓ<ΑΓ+ΑΒ ή 17,5-10<ΒΓ<17,5+10 ή 7,5cm<BΓ<27 . 5cm. ΑΔΕ ΑΔΕ ) ( ΑΔ )2 η, ( ΑΔΕ ) = ( 3 )2 = 9 )γ ι.) (( ΑΒΓ)) = λ2 η, ( ΑΒΓ ( ) ΑΒ ( ΑΒΓ ) S 25 α)

ΕΥΚΛΕΙΔΗΣ Α' 86 τ.2/29


------- -----Ομοιότητα

ίί) Από

( ΑΔΕ) = _2._ έχουμε ( ΑΒΓ) = 25 ( ΑΔΕ) οπότε 9 ( ΑΒΓ ) 25 (ΑΔΕ) - (ΑΔΕ) η, (ΑΔΕ) (ΑΔΕ) (ΑΔΕ) - 9 . "-:-:-'--( ΔΕΓΒ) ( ΑΒΓ) - (ΑΔΕ) (ΔΕΓΒ) 25 ( ΑΔΕ) -( ΑΔΕ) 16 ( ΑΔΕ) 16 9 9

2°) Δίνεται τρίγωνο ΑΒΓ. Πάνω στην πλευρά του ΑΒ δίνεται σημείο Δ και πάνω στην ΑΓ

σημείο Ε, έτσι ώστε ΜΕ = Γ = φ . Εάν ΑΕ=3 , ΕΓ=2χ+1, ΑΔ=2χ-1 και ΔΒ=χ+2, να υπολογίσετε: 1 ) Τις πλευρές ΑΒ, ΑΓ και ΑΔ των τριγώνων ΑΒΓ και ΑΔΕ. 2) Το λόγο :

(ΑΔΕ} (ΑΒΓ}

και το ΕΓ.

3) Εάν ΑΖ κάθετη στην ΔΕ και ΕΗ κάθετη στην ΒΓ, να υπολοylσετε το λόyο Λύση

�=�

.

Η

Τα τρίγωνα ΑΔΕ και ΑΒΓ έχουν Α κοινή, ΑΔΕ = Af'B = φ, άρα είναι όμοια, οπότε: ΑΔ = ΑΕ = ΔΕ ή 2χ -1 = -3- ή ( 2χ - 1) · (3χ + 1) = 3 · (2χ + 4) ή 6χ 2 -7χ - 13 = 0, ΑΒ ΒΓ 2χ + 4 3χ + 1 ΑΓ 26 = � δεκτή χ = 1 + + 12 6 Δ = (-7)2 -4·6·(-13) = 361' άρα χ1' 2 -( -7)2 ·6.J36Ϊ 712- 9 = -12 απορριπτεται ' χ2 = U 13 -1 = 1 ο . ΑΒ= 3χ + 1 = 3 ·13 + 1 1 5 ΑΓ= 2χ + 4 2 ·13 + 4 = 25 Συνεπω' ς ΑΔ= 2χ -1 = 2 · 6 3 6 2 6 2 2 10 = = 2� , ΕΓ = 2χ + ι = Ζ- 1: + Ι = 1: = � = 2) 1)

-

=

i�\ (

[) �] (Η

=

'

[) ]

!

=

-

.

3) Τα τρίγωνα ΑΔΖ και ΕΗΓ έχουν Ζ = Η 90° και ΑΔz = Af'H = φ, 10 2 2 3 = 25 . ( ΑΔΖ ) ΑΔ Άρα είναι όμοια' οπότε (ΕΗΓ) 16 64 ΕΓ 3 =

:

=

(

=

ΕΥΚΛΕΙΔΗΣ Α' 86 τ.2/30

=


Μαθηματικοί Διαγωνισμοί Επιμέλε•α: Επιτροπή Διαγωνισμών

73°ς ΠΑΝ ΕΛΛΗ Ν Ι ΟΣ ΜΑΘ ΗΤΙ ΚΟΣ ΔΙΑΓΩ Ν Ι Σ Μ ΟΣ ΣΤΑ ΜΑΘ Η ΜΑΤΙ ΚΑ 'Ό ΘΑΛΗΣ"

20

Οκτωβρίου 2012 Β· Γυ μvασίο υ

Π ρόβλη μα 1

Να υπολογίσετε την τιμή της παράστασης:

(ιs - !):

Α=

5

Λύση

3 44 - 9 5 5

·[ �6 ] 3+

11

Πρόβλη μα 2 Αν ο κ είναι πρώτος θετικός ακέραιος και διαιρέτης του μέγιστου κοινού διαιρέτη των ακεραίων 12, 30 και 54, να βρείτε όλες τις δυνατές τιμές του κ και της παράστασης: '

κ

2-2 .. Β1 κ-2 --

3

-

κ

-

2_

_

κ

Λύ ση Είναι ΜΚΔ ( 12,30, 54) = 6 . Οι θετικοί διαιρέτες του 6 είναι οι 1 , 2, 3, 6 και από αυτούς πρώτοι είναι οι 2 και 3. Άρα έχουμε κ = 2 ή κ = 3 . 3-2 2 -� 2 · 2 - _!_ _!_ - � 4 - � . Για κ = 2 έχουμε: Β - -· l·4 3 1 -3 2 _!_ 2 2 2 3-κ 3-3 -- Ο , , 2 της παραστασης 2 = - = Ο , ενω, ο διαιρετεος , , Για κ = 3 ο διαιρετης Β γινεται -κ 3 3 3- -1 2= = * Ο , οπότε η παράσταση Β δεν ορίζεται. γίνεται 3-- 2 2 -

.

_

.

_

.

--

---f- ;

t

ΕΥΚΛΕΙΔΗΣ Α' 86 τ.2/31


------- Μαθηματικοί Διαγωνισμοί ------

Πρό βλη μα 3 Ένας ελαιοπαραγωγός έχει παραγωγή λαδιού 800 κιλά. Για την καλλιέργεια του ελαιώνα του ξόδεψε 407 ευρώ και για τη συγκομιδή του καρπού από τις ελιές του ξόδεψε 1050 ευρώ. Η τιμή πώλησης του λαδιού είναι 2,5 ευρώ το κιλό και κατά την πώληση του λαδιού υπάρχουν κρατήσεις σε ποσοστό 6% πάνω στην τιμή πώλησης. (α) Να βρείτε πόσα κιλά λάδι πρέπει να πωλήσει ο παραγωγός για να καλύψει τα έξοδά του. (β) Αν επιπλέον το ελαιοτριβείο (εργοστάσιο που παράγεται το λάδι) κρατάει για την αμοιβή του το 8% του παραγόμενου λαδιού, να βρείτε πόσα κιλά λάδι θα μείνουν στον παραγωγό μετά την πώληση λαδιού για την κάλυψη των εξόδων του. Λύση

2, 5 _i_ 100 = Ο, 15 ευρώ, οπότε η καθαρή τιμή πώλησης είναι 2,5-0,15 = 2,35 ευρώ. Τα έξοδα του παραγωγού είναι 1050 + 407 = 1457 ευρώ, οπότε ο παραγωγός πρέπει να πωλήσει 1457 : 2,35 = 620 κιλά λάδι. 8 (β) Το ελαιοτριβείο θα κρατήσει 800 -- = 64 κιλά λάδι, οπότε θα μείνουν στον παραγωγό 100 + 800- ( 620 64) = 1 16 κιλά λάδι. (α) Κατά την πώληση του λαδιού οι κρατήσεις είναι

·

·

Πρόβλη μα 4

Δίνεται τρίγωνο ΑΒΓ με

Α = 60°

και

ΑΓ =

3 · ΑΒ . Παίρνουμε σημείο Ε πάνω στην 2

πλευρά ΑΓ τέτοιο ώστε ΑΕ = ΑΒ . Αν η διχοτόμος της γωνίας τμήμα ΒΕ στο σημείο Δ, να βρείτε τις γωνίες του τριγώνου ΔΕΓ. Λύση

Α

τέμνει το ευθύγραμμο

Για συντομία, θα συμβολίσουμε με α το μήκος του τμήματος ΑΒ , δηλαδή: ΑΒ = α . Εφόσον ΑΓ = � ΑΒ = � α και ΑΕ = ΑΒ = α , έχουμε:

2

Α

2

3 α ΕΓ = ΑΓ - ΑΕ = -α - α = - . 2

2

Το τρίγωνο ΑΒΕ είναι ισοσκελές ( ΑΒ = ΑΕ ) και η γωνία του Α είναι 60° , οπότε το τρίγωνο είναι ισόπλευρο και η διχοτόμος του ΑΔ είναι και διάμεσος. α Άρα είναι ΔΕ = και το τρίγωνο ΔΕΓ είναι ισοσκελές,

2

Γ

Σχήμα 1

αφού ΔΕ = ΕΓ = α . Η γωνία Ε1 είναι εξωτερική του

2

ισόπλευρου τριγώνου ΑΒΕ . Άρα έχουμε

180 Ει = 180° - ΑΕΒ = 180° - 60° = 120° , οπότε : t1 = Δ1 = Ι

0 - 1200 ° 2 = 30 .

Γ' Γυ μν ασίου

Πρό βλη μα ι Να υπολογίσετε την τιμή της παράστασης

Κ=

2 · Υ 4 · z 6 2182 , _1 , αν ειναι 3 · 13 · 2 2 · 33 + 42 · 93

(

χ

)

χ=

2-1ο , y = 4-s , z = 8-6 ,

και να αποδείξετε ότι είναι τέλειο τετράγωνο:Ρ!J.:!'Ού αριθμού. ΕΥΚΛΕΙΔΗΣ Α' 86 τ.2/32


------

Μαθηματικοί Διαγωνισμοί ---------

Λύ ση

'Εχουμε: x = 2- 1 0 , y = 4-8 = ( 22 )-8 = 2 -�-6 = ( 23 ) -6 = 2 - 18 • Ο αριθμητής του κλάσματος γίνεται: . 4 Α = χ2 • / • z6 • 2 182 = ( 2-ιο )2 . ( Τ 16) • ( Τ18 )6 • 2 182 = 2-2ο 2-64 2 -1 os • 2 1 s2 = τι ο. Ο παρανομαστής του κλάσματος γίνεται: 1 Π = 3 · ( 1 3 · 22 · 3 3 + 42 · 93 )-1 = 3 · ( 1 3 · 2 2 · 33 + 24 · 36 )- 1 = 3 · [ 22 · 3 3· ( 13 + 22 · 33 )J•

Άρα έχουμε Π ρόβλη μα 2 Να βρείτε για ποιες τιμές του πραγματικού αριθμού α οι αριθμοί 3 και -3 είναι λύσεις της ανίσωσης �J,- 5α + 2 < α(χ - 3) + 2 ( α - 1) . Λύση

Ο αριθμός 3 είναι λύση της δεδομένης ανίσωσης, αν ισχύει ότι

16 4 · 3- Sα + 2 < α ( 3- 3) + 2 (α - 1 ) � 12 - S α + 2 < 2 α - 2 � 1 6 < 7α � α > - .

7

Ο αριθμός -3 είναι λύση της δεδομένης ανίσωσης, αν ισχύει ότι

4 · (-3) - 5α + 2 < α ( -3 - 3 ) + 2 (α - 1 ) � -12 - 5 α + 2 < -6α + 2α - 2 � -8 < α � α > -8 . Επομένως οι αριθμοί 3 και -3 είναι λύσεις της ανίσωσης , όταν συναληθεύουν οι ανισώσεις 16 ' 16 α > - και α > -8 , δηλαδη όταν α > -.

7

7

Π ρόβλημα 3 Αν το εμβαδόν Ε του χωρίου ΑΒΔΓ του διπλανού σχήματος

ισούται με το

_.!.._ του εμβαδού του κυκλικού δακτυλίου που ορίζεται 12

από τους κύκλους (Ο,α) και (Ο,β) , Ο < β < α , να βρείτε τη γωνία

{

)

αι = ΑόΒ και την τιμή της παράστασης: Σ = 2ημ'αι - συν2αι ' . Λύση

Το εμβαδόν του χωρίου ΑΒΔΓ ισούται με τη διαφορά των εμβαδών των κυκλικών τομέων ( 2 Ρ2 ) ω __.:..._ .

ω ω α ΑΒ και Ο , ΓΔ , δηλαδή είναι Ε ( ΑΒΔΓ ) = πα 2 · - - πβ2 · - = ....:...._ Ο, ( ..-) { )

_

.-

2

Το εμβαδόν του κυκλικού δακτυλίου που ορίζεται από τους κύκλους (Ο, α) και (Ο, β) , Ο < β < α , ισούται με Ε ( Ο, β, α) = π ( α 2 - β2 ) , οπότε, σύμφωνα με την υπόθεση, έχουμε:

Ε ( ΑΒΔΓ )

( α 2 - Ρ2 ) ω 1 1 ω=π = � � = 12 6. Ε (Ο, β, α) 2π ( α 2 - β2 ) u

π 1 ' ημω = ημ6" = 2 και Επειδη' ειναι

συ ν

π 1 ' 2 ω = σu v 3 = 2 , εχουμε

Σ = ( 2 ημ2ω -�συv2ω )3 = (2 ( � ) 2 -�·�J3 = ( � - i) 3 = ( i) 3 = shΕΥΚΛΕΙΔΗΣ Α' 86 τ.2/33


------ Μαθηματικοί Διαγωνισμοί -----2 °ς τρόπος Η

εύρεση της γωνίας

ω

μπορεί να γίνει και με μία απλή μέθοδο των τριών, ως εξής:

Ο κυκλικός δακτύλιος εμβαδού Ε = π ( α 2 - β2 ) αντιστοιχεί σε γωνία 2π ακτινίων.

Το τμήμα του κυκλικού δακτυλίου εμβαδού Ε_ 12 αντιστοιχεί σε γωνία ω ακτινίων, οπότε έχουμε την Ε 2π =Ε

αναλογία:

12

ω

π 6

12 = - � ω = - . ω

Η τιμή της παράστασης Σ προκύπτει όπως και στον πρώτο τρόπο.

Σχήμα 2

Πρόβλη μα 4 Δίνεται ορθογώνιο ΑΒΓΔ με ΑΔ = α cm και ΑΒ < ΑΔ Η κάθετη από την κορυφή Β προς τη διαγώνιο ΑΓ την τέμνει στο σημείο Ε. Αν ισχύει ότι ΕΓ = 2 · ΑΕ , να βρείτε: (i) Το μήκος της πλευ ράς ΑΒ. (ii) Το εμβαδόν του κύκλου που περνάει και από τις τέσσερις κορυφές του ορθογωνίου ΑΒΓΔ.

.

Λύση

(i)

Έστω ΑΒ = ΓΔ = χ , AE = y, ΕΓ = 2y και BE = z . Από την εφαρμογή του Πυθαγορείου θεωρήματος στο τρίγωνα ΑΒ Ε έχουμε: χ2 = / + z2 ::::) z 2 = χ2 y 2 . (1) Από την εφαρμογή του Πυθαγορείου θεωρήματος στο τρίγωνα ΒΓΕ έχουμε: _

α 2 = 4y 2 + z 2 ::::) z 2 = α 2 - 4y2 . Από τις σχέσεις (1) και (2) λαμβάνουμε: α 2 - 4/ = χ2 - / ::::) χ2 = α 2 - 3/

(2)

(3)

Από την εφαρμογή του Πυθαγορείου θεωρήματος στο τρίγωνο ΑΔΓ έχουμε:

9y2 = χ2 + α 2 ::::) χ2 = 9y 2 - α 2 . Από τις σχέσεις (3) και (4) Σχήμα 3 aJ6 _2 _2 =α -3y2 ::::) y2 = d ::::) y =9y2 - α -, 6 6 2 if if aJ6 r if = -3 = if - 3 · = :::) Χ = a.fi . ' 6 6 2 2

(

(iii)

J

Διάμετρος του κύκλου είναι η

ΑΓ = 3y ,

οπότε η ακτίνα του είναι R = � y = a

6α 2 3πα 2 εμβαδό του κύκλου είναι Ε = πR 2 = π · = -- .

16

ΕΥΚΛΕΙΔΗΣ Α' 86 τ.2/34

8

2

(4)

έχουμε: οπότε

J6 . Το 4


------- Μαθη ματικοί ΔιαΎωνισμοi ------Ο ι λύσε ις των ασ κ ήσε ων του τεύ χους 85

Αν ο θετικός ακέραιος Α πολλαπλασιαστεί επί 3 ο ακέραιος που προκύπτει έχει το ίδιο ΝόriΌ. Αφρική, 2012 άθροισμα ψηφίων με τον Α. Να αποδείξετε ότι ο Α διαιρείται με το 9. Μ� \ Ο αριθμός 3Α διαιρείται με το 3, οπότε το άθροισμα των ψηφίων του διαιρείται με το i­ Σύμφωνα με την υπόθεση ο αριθμός Α έχει το ίδιο άθροισμα ψηφίων με τον αpιθμό 3Α , οπότε και ο Α διαιρείται με το 3. Τότε όμως ο αριθμός 3Α διαιρείται με το 9, οπότε λόγω ·Πις υπόθεσης και ο αριθμός Α διαιρείται με το 9. / / Ν8. Βρείτε όλους τους τετραψήφιους ακέραιους Α = αbcd = 1 ΟΟΟα + 1 OOb + 1 Oc + d που είναι τέτοιοι ώστε ο ακέραιος που προκύπτει μετά τη διαγραφή οποιουδήποτε ψηφίου τους διαιρεί τον αρχικό ακέραιο Α. Εσθονία, 2012 Λύ� Επειδή ο αριθμός α διαιρείται με τον τριψήφιο ακέραιο , έπεται ότι είναι d = Ο . Επειδή -αbc--0 αριθμός Α = αbcd = αbcO διαιρείται με τον ακέραιο αbd = αbΟ , έπεται ότι c = Ο . Επειδή ο αριθμός Α = αbcd = αbΟΟ διαιρείται με τον ακέραιο αcd = αΟΟ , αλλά και από τον ακέραιο bcd = bOO , έπεται ότι ο αριθμός αb = 1 Οα + b διαιρείται από το α και το b . Επομένως ο ακέραιος q, διαιρεί το b και ο ακέραιος b διαιρεί το 1 Οα , οπότε υπάρχουν ακέραιο� χ, y . τέτοιοι ώ;τε b = αχ και 1 Οα = by . Άρα έχουμε 1 Οα = axy , δηλαδή είναι xy = 1 Ο , αφού �Ρtπει να είναι α * Ο . Επομένως έχουμε τις περιπτώσεις: \ χ = 1, y = 1 Ο, οπότε α = b και έτσι προκύπτουν εννέα δυνατοί αριθμοί: 1 100, 2200, 3300,

Ν7.

4400, 5500, 6600, 7700, 8800, 9900. χ = 2, y = 5, οπότε 2α = b και έτσι προκύπτουν οι αριθμοί 1200, 2400, 3600 και 4800. χ = 5, y = 2, οπότε 5α = b και έτσι προκύπτει ο αριθμός 1 500. χ = 1 Ο, y = 1 , που είναι αδύνατη, αφού οι αριθμοί α, b πρέπει να είναι μονοψήφιοι. α Α8. Αν για τους θετικούς πραγματικούς αριθμούς α, b ισχύει ότι =::; b =::; 2α , να αποδείξετε ότι: 2 α b 5 2 =::; - + - =::; - . Αυστρlα 2012 b α 2 •

Λύ �

α b 5 α b 5 2 =::; - + - =::; - <=> Ο :::;; α2 + b2 - 2αb και -+ - =::; b α 2 b α 2 ( α - b γ � Ο (αληθής) και α + b :::;; � . b α 2 α α 1 b ' ' γιατι' απο' την υπο' θεση - :::;; b :::;; 2α , επεται Η τελευταια ότι: - :::;; - και - :::;; 2 , ' αληθευει, 2 b 2 α α b 5 , με προσ , θεση κατα, με' λη λαμβανουμε: οποτε , - + - =::; b α 2 Α9. Αν α, b, c είναι θετικοί πραγματικοί αριθμοί, να αποδείξετε ότι: α c b Νότια Αφρική 2010 1 < -- + -- + -- < 2 . α+b b+c c+α

Αφού οι

α,

b είναι θετικοί, έχουμε ότι:

Λύ ση Αφού οι α, b, c, d είναι θετικοί, έχουμε ότι: α b c -+ -- + -- > α+b b+c c+α

α b c + + -Ι α+b+c b+c+α c+α+b ·

ΕΥΚΛΕΙΔΗΣ Α' 86 τ.2/35


-------

Μαθη ματικοί Διαγωνισμοί

--------­

Διαφορετικά, η ανισότητα μπορεί να προκύψει μετά από αρκετές πράξεις, αφού είναι ισοδύναμη με την ανισότητα: ( α + b )( b + c )( c + α ) < α ( b + c )( c + α) + b ( α + b )( c + α ) + c ( α + b )( b + c ) � ( α + b )( b + c )( c + α ) - α ( b + c )( c + α) < b ( α + b )( c + α ) + c ( α + b )( b + c ) � b ( b + c )( c + α ) < b ( α + b )( c + α ) + c ( α + b )( b + c ) � b ( c + α ) ( c - α ) < c ( α + b ) ( b + c ) � bc 2 - bα2 - cb2 - cbα - αc2 - bc2 < Ο � bα 2 + cb 2 + cbα + αc 2 > Ο, που ισχύει, αφού α, b, c > Ο . Ομοίως, για τη δεύτερη ανισότητα έχουμε: α b c α+b-b b+c-c c+α-α + + -- + -- + -- < 2 � <2 α+b b+c c+α α+b b+c c+α c α b c α b � 3 - __ + __ + __ < 2 � __ + __ + __ > 1, α+b b+c c+α α+b b+c c+α η οποία αποδεικνύεται όπως και η προηγούμενη ανισότητα. AlO. Αν p, q είναι θετικοί πραγματικοί αριθμοί τέτοιοι ώστε ..!_ + ..!_ = 1 , να αποδείξετε ότι: Ρ q 1 1 1 1 1 1 (α) - :::;; + + ;;:: 1 . Ρουμανία, < - (β) 3 p (p + 1 ) q ( q + 1) 2 p (p - 1) q (q - 1 )

)

(

Λύση

' ' ' (α) Επειδη ισχυει οτι:

2012

1 1 1 1 =1 1 1 1 ' =-και - - - , - + - = 1 , εχουμε: p (p + 1) p p + 1 q (q + 1) q q + 1 p q 1 1 1 1 1 1 1 1 1 1 :::;; + + < � :::;; + < 3 p ( p + 1) q (q + 1) 2 3 p q p + 1 q + 1 2 1 1 1 1 1 p+q+2 1 1 pq - 1 1 � :::;; 1 + :::;; � :::;; < 2 � :::;; 1 - ( 3 3 p+1 q+1 p + 1) ( q + 1 ) 2 3 pq + p + q + 1 :::;; 2. 1 1 ' ' ' ' ' Αν θέσουμε p + q = s, αφου απο την υποθεση + - = 1 προκυπτει οτι p + q = pq , έπεται Ρ q ότι p + q = pq = s και η τελευταία ανισότητα γίνεται: 1 s -1 1 , . - :S; -- :S; - � 2s + 1 :S; 3s - 3 και 2s - 2 :S; 2s + 1 � s � 4 και - 2 :S; 1 (ισχυει) 3 2s + 1 2 Η ανισότητα s ;;:: 4 προκύπτει από την γνωστή ανισότητα ( p + q )2 � 4pq η οποία γίνεται s 2 � 4s � s ;;:: 4, αφού s > Ο. (β) Εργαζόμενοι ομοίως λαμβάνουμε: 1 1 _1 1 = -_ _ ..!_ + _ _ ..!_ = _1_ + _1_ _ 1 = p + q - 2 + 1 p ( p - 1) q ( q - 1 ) p - 1 p q - 1 q p - 1 q - 1 ( p - 1 )( q - 1 ) 1 1 s-2 = -- - 1 = s - 3' � + p (p - 1 ) q (q - 1) 1 οπότε αρκεί να αποδείξουμε ότι s - 3 � 1 � s � 4, που ισχύει.

(

--

J

(

J

-

Γ6. Δίνεται ορθογώνιο τρίγωνο ΑΒΓ με Α = 90" . Εξωτερικά του τριγώνου κατασκευάζουμε τετράγωνα ΑΒΔΕ και ΑΓΖΗ. Αν η ΓΔ τέμνει την ΑΒ στο Ι και η ΒΖ τέμνει την ΑΓ στο Θ, να Νότια Αφρική, 2012 αποδείξετε ότι ΑΙ = ΑΘ . ΕΥΚΛΕΙΔΗΣ Α' 86 τ.2/36


------

Μαθηματικοί Διαγωνισμοί ------

Λύση

Θέτουμε ΑΒ = r και ΑΓ = β. Τότε, από τα όμοια ορθογώνια τρίγωνα ΓΑΙ και ΓΕΔ έχουμε: ΑΙ = ΓΑ � ΑΙ ___L_ => = ΑΙ = βy . ( 1) ΕΔ ΓΕ r β+r β+r Ομοίως, από τα όμοια ορθογώνια τρίγωνα ΒΑΘ και ΒΗΖ λαμβάνουμε: ΑΘ ΒΑ => ΑΘ = Υ- => ΑΘ = βy = (2) ΗΖ ΒΗ β β+r β+r Από τις (1) και (2) λαμβάνουμε ότι ΑΙ = ΑΘ. Γ7. Σε κύκλο κέντρου Ο θεωρούμε μία διάμετρο ΑΒ και σημείο Γ πάνω στην ακτίνα ΟΒ . Η κάθετη επί την ΑΒ στο Γ τέμνει το ένα ημικύκλιο στο Δ . Κύκλος κέντρου Κ εφάπτεται στο τόξο ΒΔ και στα τμήματα ΓΔ και ΓΒ στα σημεία Ε, Ζ και Η, αντίστοιχα. Να αποδείξετε ότι το Ρουμανία, 2012 τρίγωνο ΑΔΗ είναι ισοσκελές. .

Λύση

Σχή μα 4

Επειδή οι κύκλοι c1 (0, 0Α) και c2 ( Κ, ΚΕ) εφάπτονται εσωτερικά στο Ε, έπεται ότι τα σημεία Ο, Κ και Ε είναι συνευθειακά. Επιπλέον, είναι ΚΖ 11 ΑΟ και, αν η ευθεία ΕΖ τέμνει τη ευθεία ΑΒ στο σημείο Α' , τότε θα ΕΚ ΚΖ -=έχουμε (1) ΟΑ' ΕΟ ΚΖ ΕΚ =Από τις ισότητες ΚΖ = ΕΚ και ΑΟ = ΕΟ έπεται ότι (2) ΑΟ ΕΟ Από τις ισότητες (1) και (2) έπεται ότι ΟΑ = ΟΑ', οπότε τα σημεία Α, Ζ και Ε είναι συνευθειακά. Επειδή η ΑΗ είναι εφαπτομένη του κύκλου c (Κ, ΚΕ) , από το θεώρημα δύναμης σημείου ως προς 2 κύκλο και εφαπτομένης λαμβάνουμε: (3) ΑΕ · ΑΖ = ΑΗ 2 • Επιπλέον, έχουμε ΔΕΑ = ΔΒΑ = 90° -ΔλΒ = Μr , οπότε τα τρίγωνα ΑΔΕ και ΑΔΖ είναι όμοια. Άρα έχουμε ΑΖ ΑΔ => · = (4) ΑΖ ΑΕ = ΑΔ 2 • ΑΔ ΑΕ Από τις (3) και (4) προκύπτει η ισότητα ΑΔ = ΑΗ , οπότε το τρίγωνο ΑΔΕ είναι ισοσκελές. Σχή μα Δ5. Είναι δυνατόν να aριθμήσουμε τις 1 Ο συνολικά πλευρές και διαγώνιους ενός πενταγώνου με τους δέκα ακέραιους 1 ,2 ,3 , . . . . , 1 Ο έτσι ώστε για κάθε τρίγωνο που έχει κορυφές τρεις από τις κορυφές του πενταγώνου, το άθροισμα των αριθμών που αντιστοιχούν στις πλευρές του να είναι το ίδιο; Νότια Αφρική, 2012

5

Λύση

Ας υποθέσουμε ότι το ζητούμενο είναι δυνατόν να γίνει με κοινό άθροισμα s των αριθμών που αντιστοιχούν στις πλευρές των τριγώνων. Σημειώνουμε ότι: ! = 5 = 1 0 τριγωνα με κορυφες τις κορυφες του πενταγωνου και συνολικα ορ:r ..,ονται 3 3 !· 2 ! •

,

( 5J

,

ΕΥΚΛΕΙΔΗΣ Α' 86 τ.2/37

,

,

,


------•

Μαθηματικοί Διαγωνισμοί -------

κάθε πλευρά και διαγώνιος του πενταγώνου είναι πλευρά τριών διαφορετικών τέτοιων τριγώνων.

Έτσι το συνολικό άθροισμα των αριθμών των πλευρών όλων των 1 Ο τριγώνων είναι 1 Os αλλά κάθε αριθμός από το 1 μέχρι το 1 Ο μετριέται 3 φορές, οπότε θα έχουμε: 1 Os = 3 ( 1 + 2 + 3 + ... + 1 Ο) = 1 65 => s = 1 6, 5 που είναι άτοπο, γιατί ο s είναι θετικός ακέραιος. Δ6. Στο Καρτεσιανό επίπεδο θεωρούμε το σύνολο S = { (χ, y ) : χ, y ε Ζ} όλων των σημείων του με ακέραιες συντεταγμένες. Να βρείτε τον ελάχιστο αριθμό χρωμάτων που είναι αναγκαία για το χρωματισμό όλων των σημείων του S έτσι ώστε να μην υπάρχουν δύο σημεία του S με Εσθονία, 2012 απόσταση 5 που να έχουν χρωματιστεί με το ίδιο χρώμα. ,

·

Παρατηρούμε ότι 2 χρώματα Α και Β είναι αρκετά. Αυτό διαπιστώνεται, αν χρωματίσουμε όλα τα σημεία με άθροισμα συντεταγμένων άρτιο με το χρώμα Α και όλα τα υπόλοιπα με το χρώμα Β. Όλα τα σημεία που απέχουν απόσταση 5 μονάδων από το σημείο με συντεταγμένες M (x, y ) είναι της μορφής: (x ± 4, y ± 3 ) , (x ± 3, y ± 4), (x ± 5, y ), (x, y ± 5 ) . Όμως σε κάθε περίπτωση το άθροισμα των συντεταγμένων των παραπάνω σημείων είναι άρτιο, αν χ + y περιττός, και είναι περιττό, αν χ + y άρτιος. Άρα όλα τα σημεία που απέχουν από το σημείο Μ (χ, y ) απόσταση 5 μονάδων χρωματίζονται με διαφορετικό χρώμα από το σημείο M (x, y ) . Λύ ση

Ασ κ ήσε ις γ ια λύσ η All.

Να βρείτε τον αριθμό των ψηφίων του αριθμού Α = 5 1 40 • 847 •

Α12. Να αποδείξετε ότι για κάθε

� χ ε JR αληθεύει η ανισότητα: χ6 + χ4 - χ3 - χ + > Ο. 4

x, y είναι πραγματικοί αριθμοί διαφορετικοί μεταξύ τους και διαφορετικοί από το yz - χ2 zx - y2 yz - χ2 zx - y 2 = χ + y + z. = , να αποδει'ξετε οτι και ισχύει ότι: 1-χ 1-y 1-χ 1-y Α1 3 . Αν οι

,

=

Α14. Να αποδείξετε ότι, για κάθε

Γ8 .

Δίνεται τετράγωνο PC = ..[3. Να βρείτε:

1

x, y > Ο , ισχύει ότι: 1 �

(χ + y ) ( x3 + / ) 9 2 � -8 . 2 2 ) + y (χ

και σημείο Ρ στο εσωτερικό έτσι ώστε ΡΑ = 1 , ΡΒ = .J2 και το μήκος PD , 2 .) το μέτρο της γωνίας ΑΡΒ .

ABCD

1.)

Δίνεται τρίγωνο ABC με Α = 90·. θεωρούμε σημεία D και Ε πάνω στα ευθύγραμμα τμήματα AC και BD , αντίστοιχα, έτσι ώστε ABc=ECn=cEn. Να αποδείξετε ότι: ΒΕ = 2 AD . Ν9. Είναι δυνατόν η περίμετρος ενός τριγώνου του οποίου τα μήκη των πλευρών είναι ακέραιοι να διαιρείται με το διπλάσιο της μεγαλύτερης πλευράς του; Γ9.

·

Nl Ο. Βρείτε

όλες τις τριάδες ( m, n, p ) , όπου

m, n,

p

θετικοί ακέραιοι και

5 m + 2n Ρ , , , , τετοιες ωστε ο αριθμος m 2n Ρ , ειναι τετραγωνο ακεραιου. 5 .

,

_

ΕΥΚΛΕΙΔΗΣ Α' 86 τ.2/38

,

p πρώτος, που είναι


ΕΛΛΗΝ Ι ΚΗ ΜΑΘΗΜΑΤΙ ΚΗ ETAI PEIA ΠΑΡΑΡΤΗΜΑ Ν. ΜΑΓΝΗΣΙΑΣ

1 ος ΤΟΠΙΚΟΣ ΔΙΑΓΩΝΙΣΜΟΣ ΓΙΑ ΜΑΘΗΤΕΣ Α ΊΥΜΝΑΣΙΟΥ ΣΤΑ ΜΑΘΗΜΑΤΙΚΑ ΣΑΒΒΑΤΟ 1 9 NOEMBPIOY 201 1

Επιτροπή Θεμάτων : Πέτρος Νικολουδάκης, Σπύρος Δημόπουλος, Αναστασία Κοντονίνα ΘΕΜΑΤΑ ΚΑΙ ΛΥΣΕΙΣ 1° ΘΕΜΑ : 1 , 4 , 0.33 , 2 9 5 3 α) Ποιος είναι ο μικρότερος; β) Ποιος είναι ο μεγαλύτερος; γ) Να τους κατατάξετε κατά αύξουσα διάταξη. δ)Ποιος είναι ο μέσος όρος του μικρότερου και του μεγαλύτερου; Δίνονται οι παρακάτω επτά αριθμοί :

..!,

4,

0.3 ,

0.55

Λύση

Μονάδες 5

ι ος τρόπος. Μετατρέπω τα κλάσματα σε δεκαδικούς, κάνοντας τις διαιρέσεις αριθμητής διά .!.. =ο 5 i =o'444 ' ο . 33 ' 0.3 , 0.55 . Ή παρονομαστη'ς. .3. =ο'666' i =o' 8 3 2 ' 9 5 2°ς τρόπος. Μετατρέπω τους δεκαδικούς σε ομώνυμα κλάσματα. .3. = 600 i = 225 ' _!_ = 450 ' i = 400 ' ο . 33= 33 = 297 ' ο . 3 = i._ = 270 ' 0.55= 55 = 495 . ' 5 900 10 900 2 900 100 900 9 900 100 900 3 900 Οπότε: α) μικρότερος είναι ο 0,3 β) μεγαλύτερος είναι ο i = 0,8 5 γ) κατάταξη κατά αύξουσα διάταξη : 0,3 ( 0,33 (

i ( _!_ ( 0,55 ( .3. ( i . 2

9

3 5 11 ιι 0' 3 + i i._ + !_ 5 10 1 0 10 10 ι 1 δ) ο μέσος όρος του μικρότερου και του μεγαλύτερου είναι : 2 2 2 2 20 ι _..:::...

_

2° ΘΕΜΑ : α) Να βρείτε τους φυσικούς αριθμούς που διαιρούμενοι με τον αριθμό δίνουν πηλίκο ίσο με το 4 υπόλοιπο της διαίρεσης του αριθμού 1 1 1 με τον αριθμό 13 και υπόλοιπο τους πρώτο φυσικό αριθμό. Λύση

Κάνουμε την διαίρεση 1 1 1 : 1 3 και έχουμε πηλίκο 8 και υπόλοιπο 7. Ο φυσικός αριθμός που ζητάμε θα είναι : Διαιρετέος = (διαιρέτης) Χ (πηλίκο) + (υπόλοιπο) Άρα Δ = 4 Χ 7 + υ, όπου το υ = 2, ή 3. Επομένως Δ = 28+2 = 30, ή Δ = 28 + 6 = 3 1 β) Να κάνετε τις παρακάτω πράξεις και να aπλοποιήσετε τα (τελικά) κλάσματα : 1 1 3 2 �:2 - +1 +� 1 ι 1 ι 4 3 -........ 3) 2) τι = ........ 4) = ........ 5) (- --) : (- : -) = ........ ι) 2 3 2 3 8 4 -·2 2� - 1� 3- : 19 4 4 6 4

Λύση

1 1 3 2 5 -++6 ι ) 2 3 _Q__Q_ = 9 1 1 2 22 7 � .!.. 2 -ι 4 6 4 6 12 12

- - -

f-

5 6 5 · 12 =2 = 5 5·6 12 ΕΥΚΛΕΙΔΗΣ Β ' 86 τ.2/39

Μονάδες 5


-------

lο; ΤΟΙΠΚΟΣ ΔΙΑΓΩΝΙΣΜΟΣ ΠΑ ΜΑΘΙΠΕΣ Α' ΓΥΜΝΑΣΙΟΥ ΣΤΑ ΜΑΘΙΙΜΑΠΚΑ ----_,:....--

3° ΘΕΜΑ : Ο Κώστας κάνει ένα συγκεκριμένο κυκλικό στίβο, τρέχοντας, σε 4 λεπτά, ο Σπύρος κάνει την ίδια διαδρομή σε 5 λεπτά και ο Πέτρος σε 6 λεπτά. Αν ξεκινήσουν και οι τρεις ταυτόχρονα από ένα σημείο Α του κυκλικού στίβου τότε : 1) Μετά από πόσο χρόνο θα βρεθούν στο ίδιο σημείο Α ταυτόχρονα και οι τρεις; 2) Πόσους κύκλους θα διαγράψει ο καθένας σ' αυτό το χρονικό διάστημα; 3) Μέσα σ' αυτό το χρονικό διάστημα, που χρειάζεται για να συναντηθούν και οι τρεις στο σημείο Α, ποιο από τα ζευγάρια (Κώστας, Σπύρος), (Κώστας, Πέτρος), (Σπύρος, Πέτρος) συναντήθηκαν τις περισσότερες φορές στο σημείο Α και πόσες φορές το κάθε ζευγάρι; Μονάδες 5 Λύση

1) Ε.Κ.Π. (4,5,6)=60 . Μετά από 60 λεπτά συνάντηση και των τριών στο σημείο Α. 2) Κώστας 60:4 = 1 5 κύκλους Σπύρος 60:5 = 12 κύκλους Πέτρος 60: 6 = 1 0 κύκλους 3) (Κώστας, Σπύρος) : Ε.Κ.Π. (Κώστας, Σπύρος)=Ε.Κ.Π.(4,5)=20, Άρα 60:20 = 3 συναντήσεις (Κώστας, Πέτρος) : Ε.Κ.Π. (Κώστας, Πέτρος)=Ε.Κ.Π.(4,6)=12, Άρα 60: 1 2 = 5 συναντήσεις (Σπύρος, Πέτρος) : Ε.Κ.Π. (Σπύρος, Πέτρος)=Ε.Κ.Π.(5,6)=30, Άρα 60:30 = 2 συναντήσεις "··

4° ΘΕΜΑ : Δίνεται ένα ορθογώνιο παραλληλόγραμμο ΑΒΓΔ με μήκος ΑΒ 7 εκ., ένα ισόπλευρο τρίγωνο ΕΖΗ και ένα τετράγωνο ΘΙΚΛ, τα οποία έχουν την ίδια περίμετρο, ίση με 24 εκ. Να υπολογίσετε : α) Το πλάτος του ορθογωνίου παραλληλογράμμου, την πλευρά του ισοπλεύρου τριγώνου και την πλευρά του τετραγώνου. =

Λ

Δ

Γ

ΒΓ = ;

7 εκ.

Α

Β

z

ΖΗ Μ

;

Η

θ

θl = ;

β) Τα εμβαδά του ορθογωνίου παραλληλογράμμου και του τετραγώνου. γ) Το ύψος του ισοπλεύρου τριγώνου, αν γνωρίζουμε ότι έχει εμβαδόν 27,72 τετ. εκ. Μονάδες 5

Λύση

α) Το πλάτος του ορθογωνίου παραλληλογράμμου: 24: 2 = 1 2, 1 2-7 = 5 εκ πλευρά του ισοπλεύρου τριγώνου : 24 : 3 = 8 εκ πλευρά του τετραγώνου : 24 : 4 = 6 εκ β) Εμβαδόν του ορθογωνίου παραλληλογράμμου : πλευράΧπλευρά = 7Χ5 = 35 τετ. εκ. και Εμβαδόν του τετραγώνου = 6Χ6 = 36 τετ.εκ. γ) Το ύψος του ισοπλεύρου τριγώνου : επειδή ΕμβΤριγ=(βάσηΧύψος):2 έχουμε 27,72Χ2:8 = 6,93 εκ. •

1

ΕΥΚΛΕΙΔΗΣ Β' 86 τ.2/40


Νέες τεχνολογίες κα ι Μαθηματ ι κά. ======

Σταμάτης Μακρής

Εισαγωγικά 1) Η GeoGebra είναι ένα λογισμικό δυναμικής Γεωμετρίας.

Είναι ελεύθερη και πολλών πλατφόρμων εφαρμογή μαθηματικών για όλα τα επίπεδα της εκπαίδευσης. Η διαδικτυακή της διεύθυνση είναι: http://www.geogebra.org Για να την εγκαταστήσετε στον υπολογιστή σας πηγαίνετε στη σελίδα της και επιλέξτε «Λήψη Αρχείων». Κατόπιν επιλέξτε το «πρόγραμμα εγκατάστασης χωρίς σύνδεση» και δώστε την επιλογή του λειτουργικού σας συστήματος. Στο αναδυόμενο μενού επιλέξτε αποθήκευση και θα σας αποθηκεύσει ένα αρχείο(π.χ. «GeoGebra-Windows-Installer-4-0-36-0»). Με διπλό κλικ στο αρχείο θα ξεκινήσει η διαδικασία εγκατάστασης. Όταν η εγκατάσταση ολοκληρωθεί, θα υπάρχουν στη επιφάνεια εργασίας του υπολογιστή σας δύο εικονίδια· ένα της Geogebra και ένα της GeoGebraPrim που είναι έκδοση για χρήση όταν είστε συνδεδεμένοι στο διαδίκτυο. 2) Ανοίγοντας ένα νέο αρχείο GeoGebra θα δείτε το παρακάτω: l= Ι ΙΞJ I

Ο GeoGebrιι Αρχείο Επειεpγασία Προβολή Όψεις Επιλογές Εργαλ!ία Παράθυρο Βοήθεια

[§JΙ 00lJJ Ι [E�[QJI �[ΣIΞJ�I C±J Άλγεβρα

(Ξ)@J0

Ελεύθερα αvτικείμι.να Ειαρτημένα αvτικείμι.να

� � G�

Μετακfvηση

Γραφικά

Υ

2

1

D

χ

ο

2

1

3

4

� Η πρώτη γραμμή έχει διάφορα μενού. Από το μενού «Αρχείο» με «Αποθήκευση ως» δώστε ένα όνομα στο αρχείο που θα δημιουργήσετε ( πχ.Gonies το πρόγραμμα θα βάλει μόνο του κατάληξη .ggb) Σώστε το αρχείο σε όποιο φάκελο θέλετε για να το βρίσκετε εύκολα. 4 !τη δεύτερη γραμμή βρίσκονται τα Εργαλεία Γεωμετρίας. Οδηγίες για τα «Εργαλεία Γεωμετρίας»

� Μπορείτε να ενεργοποιήσετε ένα εργαλείο με αριστερό «Click» στην αντίστοιχη εικόνα. � Μπορείτε να ενεργοποιήσετε την ομάδα εργαλείων πατώντας στο κόκκινο βελάκι στην κάτω δεξιά γωνία. � Στο τέλος της γραμμής εργαλείων εμφανίζονται οδηγίες για τη χρήση του εργαλείου που έχετε επιλέξει. � Ακόμη βλέπετε δύο παράθυρα αριστερά της «Άλγεβρας» και δεξιά της γεωμετρίας «Γραφικά». ΕΥΚΛΕΙΔΗΣ Α'

86 τ.2/41


------

Νέες τεχνολογίες και Μαθηματικά.

--------­

Παραπληρωματικές γωνίες

Π ροετοιμασία �

Ανοίξτε ένα αρχείο GeoGebra

Κλείστε το παράθυρο της «Άλγεβρας» από το μενού: Προβολή � Άλγεβρα � Αποκρύψτε τους άξονες από το μενού: Προβολή � Άξονες �

Από το μενού Επιλογές επιλέξετε «Ετικετοποίηση αντικειμένων» «Μόνο σε νέα σημεία»

Οδηγίες Βήμα-Βήμα

ιο

2

0

30

40

so

[CJ � [2] � [Ζ] � � � [2] �

Το εργαλείο «Ημικύκλιο που περνά από δύο σημεία» θα το βρείτε στην 6η στοίβα των γεωμετρικών εργαλείων. Όταν το επιλέξετε με κλικ στο χώρο της γεωμετρίας, θα εμφανιστεί ένα σημείο Α και με δεύτερο κλικ δεξιότερα ένα σημείο Β και θα ορίσετε το ημικύκλιο δια,μέτι:>_ου ΑΒ. Επιλέξτε το εργαλείο της μετακίνησης για να αλλάξετε τις θέσεις των σημείων Α και Β στο παράθυρο των Γραφικών. Από την 3η σειρά εργαλείων επιλέξτε το 2ο «Τμήμα μεταξύ δύο σημείων». Με κλικ στο σημείο Α και μετά στο Β θα δημιουργήστε το ευθύγραμμο τμήμα ΑΒ. Επιλέξτε το εργαλείο της μετακίνησης. Αν φέρετε το δείκτη στο ευθύγραμμο τμήμα ΑΒ, κρατώντας πατημένο το αριστερό κλικ, ο δείκτης θα γίνει χεράκι και μπορείτε να αλλάξετε τη θέση του τμήματος ΑΒ. Το εργαλείο «Μέσο ή κέντρο» θα το βρείτε στη 2η στοίβα των γεωμετρικών εργαλείων. Όταν το επιλέξετε, με κλικ στο ευθύγραμμο τμήμα ΑΒ θα δημιουργήστε το μέσο Γ του τμήματο_ς ΑΒ. Επιλέξτε το σημείο Γ και με δεξί κλικ επιλέξτε «Μετονομασία» και στο παράθυρο διαλόγου πληκτρολογήστε Ο. Έτσι το μέσον του τμήματος ΑΒ θα είναι το σημείο Ο. Από τη 2η σειρά εργαλείων επιλέξτε το 1 ο «Νέο Σημείο» και κάντε κλικ πάνω στο ημικύκλιο ΑΒ. Θα δημιουργήσετε το σημείο Δ. Επιλέξτε το εργαλείο της μετακίνησης και κατόπιν μετακινήστε το σημείο Δ. Θα δείτε ότι μετακινείται μόνο επάνω στο ημικύκλιο. Από την 3η σειρά εργαλείων επιλέξτε το 2ο«Τμήμα μεταξύ δύο σημείων». Με κλικ στο σημείο Δ και μετά στο Ο θα δημιουργήσετε το ευθύγραμμο τμήμα ΟΔ. Επιλέξτε το εργαλείο της μετακίνησης και κατόπιν μετακινήστε το σημείο Δ. Θα δείτε ότι μετακινείται μαζί και το τμήμα ΟΔ.

ΕΥΚΛΕΙΔΗΣ Α' 86 τ.2/42


------

Νέες τεχνολογίες και Μαθη ματικά.

-------

Από την 8η σειρά εργαλείων επιλέξτε το 2ο «Γωνία». Μετά την επtλογή του εργαλείου, επιλέξτε τα σημεία Β , Ο και Δ με αυτή την σειρά. Θα δημιουργήσετε τη γωνία α = ΒΟΔ. Το πρόγραμμα δίνει στις γωνίες πράσινο χρώμα, αΜά μπορείτε να το αλλά ετε δε ί κλικ σ ωνία στι «lδιό τε » στο «Χ ώ α». Επιλέξτε το εργαλείο της μετακίνησης και κατόπιν μετακινήστε το σημείο Δ. Θα δείτε ότι ωνία σα παί νει τι έ από 0° έω 1 80° Από την 8η σειρά εργαλείων επιλέξτε το 2ο «Γωνία». Μετά την επtλογή του εργαλείου επιλέξτε τα σημεία Δ , Ο και Α με αυτή την σειρά. Θα δημιουργήσετε τη γωνία β = ΔΟΑ. Δώστε σ ωνία ΔΟΑ δια ο ετικό ώ α από ν ΒΟΔ. Επιλέξτε το εργαλείο της μετακίνησης και κατόπιν μετακινήστε το σημείο Δ. Θα δείτε ότι οι ωνίε αλλά συν έτ ο. Με δεξί κλικ στο ημικύκλιο ΑΒ από το μενού «Iδιότητες» επιλέξτε «Στυλ» και αΜά τε το στυλ α ' του ικυκλίου σε διακεκο ένε τελίτσε . Από τη 10η σειρά εργαλείων επιλέξτε το 1 ο «Εισαγωγή κειμένου». Κάντε κλικ σε ελεύθερο χώρο αριστερά από το σχήμα σας και στο αναδυόμενο μενού Κείμενο στην «Επεξεργασία» πληκτρολογήστε Γωνία. Κατόπιν από το Αντικεί ενο επιλέ τε ωνία α και δώστε ΟΚ. Επιλέξτε το εργαλείο της μετακίνησης και κατόπιν μετακινήστε το σημείο Δ. Θα δείτε ότι το κείμενο που μόλις δημιουργήσατε είναι δυνα ικό και ,όποτε αλλά ει τι ' ωνία, δεί ει ν καινού ια τι Από τη 10η σειρά εργαλείων επιλέξτε το 1 ο «Εισαγωγή κειμένου». Κάντε κλικ σε ελεύθερο χώρο αριστερά από το σχήμα σας και στο αναδυόμενο μενού Κείμενο στην «Επεξεργασία» πληκτρολογήστε Παραπληρωματική. Κατόπιν από το «Αντικείμενο» επιλέξτε τη γωνία β και δώστε ΟΚ. Επιλέξτε το εργαλείο της μετακίνησης και κατόmν μετακινήστε το σημείο Δ. Θα δείτε ότι το κείμενο που μόλις δημιουργήσατε είναι δυναμικό και, όποτε αλλάζει τιμή η γωνία, δείχνει την καινούργια τιμή της πα απλ ω ατι ' . Σημειώστε ότι: -+

Μπορείτε με δεξί click και επιλέγοντας «Iδιότητες» σε οποιοδήποτε αντικείμενο να αλλάξετε χρώματα και το στυλ. Φτιάξτε τα αντικείμενα με χρώματα της αρεσκείας σας.

-+

Αν δεν θέλετε τα δεκαδικά ψηφία στο μέτρο της γωνίας, μπορείτε από το μενού «Επtλογές» να επιλέξετε «Στρογγυλοποίηση σε» Ο δεκαδικές θέσεις.

Ερωτήματα προς διερεύνηση : 1)

2) 3) 4)

Τι συμβαίνει όταν α=0° ή α= 1 80°; Για ποια θέση του σημείου Δ η γωνία είναι ίση με την παραπληρωματική της; Για ποια θέση του σημείου Δ η γωνία είναι διπλάσια της παραπληρωματικής της; Τι αλλάζει στα παραπάνω ερωτήματα, αν φέρετε σε κατακόρυφη θέση τα σημεία Α και Β; ΕΥΚΛΕΙΔΗΣ Α' 8 6 τ.2/43


«Χαίρομαι να λύνω»

Ένα από τα πρώτα και γνωστότερα προβλήματα της θεωρίας γραφημάτων είναι ο σχεδιασμός ενός γραφήματος με μια μόνο γραμμή, δηλαδή με τέτοιο τρόπο ώστε να μη σηκώσει κάποιος το μολύβι του κατά τη σχεδίαση αλλά και να μην ξαναπεράσει από την ίδια γραμμή. Τα γραφήματα είναι σύνολα από κορυφές και πλευρές. Οι κορυφές είναι τα σημεία εκείνα όπου διασταυρώνονται οι πλευρές και οι πλευρές είναι οι δεσμοί ανάμεσα σε ζεύγη κορυφών. Αν ξεκινήσουμε από μια κορυφή ενός γραφήματος και διαδο­ χικά εvαλλάσουμε πλευρές και κορυφές ώσπου να φτάσου­

Λέοναρντ Όιλερ (1 707-1 783)

με σε μια άλλη κορυφή του γραφήματος, τότε η διαδρομή που ακολουθήσαμε ονομάζεται δρόμος. Αν σε ένα δρόμο κάθε πλευρά του γραφήματος εμφανίζεται μια μόνο φορά, τότε ο δρόμος λέγεται δρομίσκος. Ένας δρομίσκος που διέρχεται από όλες τις πλευρές του γραφήματος ονομάζεται

δρομίσκος- Όιλερ,

προς τιμή του Όιλερ, ο οποίος ασχολή­

θηκε πρώτος με τέτοιου είδους προβλήματα στο έργο του

"Επτά Γέφυρες τοu Κένιγκσμπεργκ ".

Έστω το παραπάνω γράφημα, το οποίο είναι γνωστό και σαν "σπίτι του Άγιου Βασίλη " από ένα παλιό γερμανι­ κό γρίφο για παιδιά. Σκοπός είναι να σχεδιάσουμε το σπί­ τι χωρίς να σηκώσουμε το μολύβι και χωρίς να σχεδιά­ σουμε πάνω από μια φορά την ίδια πλευρά. Δηλαδή, σκο­ πός μας είναι να βρούμε ένα δρομίσκο-Όιλερ. Αν ξεκινήσουμε από την κορυφή D για παράδειγμα, έ­ νας δρομίσκος-Όιλερ αποτελείται από τις κορυφές D, C, Β, Α, C, Ε, Β, D, Ε.

ΕΥΚΛΕΙΔΗΣ Α' 86 τ.2/44

Ελβετός μαθηματικός και φυσικός, με μεγάλη συνεισφορά σε πολλούς τομείς των μαθηματικών όπως η γεωμετρία, η τριγωνομετρία, ο διαφορικός και ολο­ κληρωτικός λογισμός, και η θεωρία α­ ριθμών. Υπήρξε ένας απο τους πιο πα­ ραγωγικούς μαθηματικούς όλων των εποχών, αφού το δημοσιευμένο έργο του περιλαμβάνει 886 άρθρα και βιβλία, και μπορεί να γεμίσει περίπου 90 τό­ μους, δηλαδή σχεδόν 45.000 σελίδες. Παρά το γεγονός ότι ήταν μισότυφλος το μεγαλύτερο μέρος της ζωής του και εντελώς τυφλός τα τελευταία 1 7 χρόνι­ α, η ασυνήθιστη, φωτογραφική του μνήμη και οι θρυλικές μαθηματικές του ικανότητες τον βοήθησαν να φέρει εις πέρας παραπάνω απο το μισό του έργο μέσα σε αυτό το διαστήμα. Το έργο του είναι τόσο σημαντικό που είναι πρακτι­ κά αδύνατο να ασχοληθεί κάποιος με ένα τομέα των Μαθηματικών, χωρίς να συναντήσει το όνομα του. Στα πιο γνω­ στά τα έργα του περιλαμβάνονται: " Ε ι­ σαγωγή στην ανάλυση των aπε ίρως μικρών" ( 1 748), " Γενικές αρχές του

διαφ ορικού λογισμού" ( 1 755), " Γ ενι­ κές αρχές του ολοκληρωτικού λογι­

σμού" ( 1 768- 1 774). Η εργασία του πά­ νω στις " Επτά Γέφυρες του Κ ένιγκσ­ μπεργκ" που δημοσιεύτηκε το 1 736 θεωρείται το πρώτο άρθρο στην ιστορία της Θεωρίας Γραφημάτων.


------ Χαίρο μαι να λύνω

------

Α ν σε ένα γράφημα υπάρχει ένας κλειστός δρομίσκος­ Όιλερ, δηλαδή αν ξεκινήσουμε από μι α κορυφή, δι ασχίσου­ με κάθε πλευρά ακριβώς μι α φορά και καταλήξουμε στην ί­ δι α κορυφή, τότε το γρά φημα λέγετα ι γρά φημα-Όιλερ.

Ας θεωρήσουμε τα παρακάτω γραφήματα:

Ο ι Γ έ φυρες του Κ ένιγκ σ μ περγ κ

Στο πρώτο γράφημα παρατηρούμε ότι από οποιαδήποτε κορυφή και να ξεκινήσουμε μπορούμε να επιστρέψουμε σε αυτή περνώντας ακριβώς μια φορά απο κάθε πλευρά. Για παράδειγμα, αν ξεκινήσουμε απο την κορυφή Α, ένας κλειστός δρομίσκος-'Οιλερ είναι ο: Α, Β, C, F, Ε, D, Β , Ε, Α. Το γράφημα αυτό είναι επομένως γράφημα-'Οιλερ. Αν δοκιμάσουμε τώρα να βρούμε κάποιο κλειστό δρομίσκο­ Όιλερ στο δεύτερο γράφημα διαπιστώνουμε ότι όχι μόνο δεν υπάρχει, αλλά και ότι είναι αδύνατο να ξεκινήσουμε από μια κορυφή και να φτάσουμε σε οποιαδήποτε άλλη του γραφήματος χωρίς να αποφύγουμε να περάσουμε δεύ­ τερη φορά από κάποια πλευρά. Δεν υπάρχει δηλαδή ούτε δρομίσκος-'Οιλερ. Στο ερώτημα πότε ένα γράφημα είναι γράφημα-'Οιλερ και πότε ένα γράφημα έχει δρομίσκο-'Οιλερ, δίνει απάντη­ ση η πρόταση: •

Ένα γράφημα είναι γράφημα-'Όιλερ αν από κάθε κορυφή του διέρχεται άρτιος αριθ μός πλευρών. Ένα γράφημα έχει δρομίσκο-'Όιλερ αν έχει το πολύ δύ ο κορυφές από τις οποίες διέρχεται περιττός α­ ριθ μός πλευρών.

Πα ρατήρηση l η Στο πρώτο γράφημα παρατηρούμε ότι από κάθε κο­ ρυφή διέρχεται άρτιος αριθμός πλευρών. Για καθεμiα από τις κορυφές Α, C, D, F διαπιστώνουμε ότι διέρχο­ νται δύο πλευρές, ενώ από την κορυφή Ε, όπως και την Β διέρχονται τέσσερις πλευρές. Επομένως το πρώτο γράφημα εiναι γράφημα-'Οιλερ. Στο δεύτερο γράφημα παρατηρούμε ότι έχουμε μόνο δυο κορυφές με άρτιο αριθμό πλευρών (τις κορυφές D και F), ενώ οι υπόλοιπες τέσσερις κορυφές έχουν περιτ­ ΕΥΚΛΕΙΔΗΣ Α' 86 τ.1/45

Την πόλη του Κένιγκσμπεργκ (που όσο ζούσε ο Όιλερ άνηκε στη Πρωσία και σήμερα ανήκει στη Ρωσία) διασχίζει ο ποταμός Πρέ­ γκελ, με τέτοιο τρόπο ώστε να χω­ ρίζεται η πόλη σε τέσσερις περιο­ χές, σχηματίζοντας δυο νησάκια στο κέντρο της. Οι κάτοικοι είχαν κατασκευάσει επτά γέφυρες για να έχουν πρόσβαση σε όλα τα σημεία της πόλης. Το να ξεκινήσει κάποιος απο ένα σημείο της πόλης και να επιστρέψει σε αυτό, διασχίζοντας όλες τις γέφυρες απο μια φορά α­ κριβώς, ήταν ένα άλυτο πρόβλημα για τους κατοίκους της περιοχής. Ο γρίφος προσέλκυσε και την προσο­ χή του Όιλερ. Ο Όιλερ παρατήρησε ότι αν και μπορεί κάποιος να δοκι­ μάσει όλες τις πιθανές διαδρομές ως τρόπο λύσης, το πρόβλημα πε­ ριπλέκεται στη περίπτωση που αυ­ ξάνεται ο αριθμός των γεφυρών (τι θα γινόταν για παράδειγμα αν είχα­ με είκοσι γέφυρες;). Έτσι, σκέφτη­ κε ένα τρόπο να απλοποιήσει το πρόβλημα. Η λύση που πρότεινε ήταν η αρχή για μια νέα Γεωμετρία όπου οι αποστάσεις δε παίζουν κα­ νένα απολύτως ρόλο. Καταρχάς, aντιστοίχισε τις τέσσερις περιοχές γης που χώριζε ο ποταμός με τέσ­ σερα σημεία (κορυφές) τα οποία και ονόμασε με κεφαλαία γράμμα­ τα. Στη συνέχεια ένωσε με γραμμές (πλευρές) τις περιοχές που επικοι­ νωνούσαν μεταξύ τους με γέφυρες. Το πρόβλημα είχε τώρα τη παρακά­ τω απλή μορφή.


------

Χαίρομαι να λύνω

-------

τό αριθμό πλευρών. Έτσι, το δεύτερο γράφημα δεν είναι γράφημα-'Όιλερ αλλά ούτε περιλαμβάνει δρομίσκο­ 'Όιλερ. Τέλος, το "σπίτι του Άγιου Βασίλη" αν και δεν είναι γράφημα-'Όιλερ, περιλαμβάνει δρομίσκο-'Όιλερ αφού ακριβώς δύο κορυφές του έχουν περιττό αριθμό πλευ­ ρών. r

Άσκηση lη Να βρείτε ποιά από τα παρακάτω γραφήματα είναι γραφήματα-'Όιλερ και ποια περιλαμβάνουν δρομίσκους­ 'Όιλερ .

�I �

Λύση

Στο πρώτο γράφημα κάθε κορυφη εχει άρτιο αριθμό πλευρών, άρα είναι γράφημα:Οιλερ με κλειστό δρομίσκο: A, C, E, D, B, E, A .

Το δεύτερο γράφημα έχει τέσσερις κορυφές απο τις ο­ ποίες διέρχεται περιττός αριθμός πλευρών, επομένως δεν είναι γράφημα:οιλερ, ούτε περιλαμβάνει δρομίσκο-Όιλερ. Το τρίτο γράφημα έχει ακριβώς δύο κορυφές με περιττό αριθμό πλευρών, τις Α, Β. Ένας δρομίσκος:οιλερ είναι ο: A, B, D, A, C, D. Παρατήρηση 2η Για να προσδιορίσουμε το κλειστό δρομίσκο σε ένα γράφημα-'Όιλερ μπορούμε να χρησιμοποιήσουμε τον πα­ ρακάτω αλγόριθμο, που είναι γνωστός ως αλγόριθμος του Fleury :

Ο

συλλογισμός του Όιλερ ήταν ο εξής. Κατάλαβε ότι σε οποιο­ δήποτε απο τα τέσσερα κομμά­ τια γης της πόλης του Κένιγκσ­ μπεργκ και αν βρίσκεται κανείς, θα έπρεπε να έχει μια γέφυρα για να μπορεί να φύγει και μια γέφυρα για να μπορεί να επι­ στρέψει. Αυτό που χρειάζεται λοιπόν είναι ένας άρτιος αριθ­ μός γεφυρών. Στο Κένιγκσ­ μπεργκ όμως, κάθε περιοχή είχε περιττό αριθμό γεφυρών. Ο πε­ ρίπατος επομένως των κατοί­ κων της πρωσικής πόλης ήταν αδύνατος ! Η λογική του Όιλερ δεν έδωσε όμως απλά μια απά­ ντηση σε ένα πρόβλημα που απασχολούσε μια μικρή κοινω­ νία ανθρώπων πριν απο τρεις αιώνες. Η λύση στο γρίφο των γεφυρών του Κένιγκσμπεργκ έθεσε τη βάση για μια νέα θεω­ ρία των Μαθηματικών που θα μπορούσε να εξηγήσει απο τον τρόπο που βλέπουμε στη Χη­ μεία τα άτομα μέχρι και τη κα­ τανόηση του διαδικτύου, τη Θεωρία Γραφημάτων.

1. Επιλέγουμε οποιαδήποτε κορυφή του γραφήματος για να ξεκινήσουμε. 2. Από την κορυφή αυτή επιλέγουμε την πλευρά που θα

διασχίσουμε και στη συνέχεια τη διαγράφουμε, ώστε να μην την επιλέξουμε ξανά.

3. Δεν επιλέγουμε μια πλευρά η διαγραφή της οποίας μπορεί να χωρίσει το γράφημα μας

σε δυο κομμάτια, εκτός και αν δεν μπορούμε να κάνουμε αλλιώς. ΕΥΚΛΕΙΔΗΣ Α' 86 τ.2/46


------

Χαίρομαι να λύνω ------

Άσκη ση 2η Να βρείτε το κλειστό δρομίσκο στο γράφημα-'Όιλερ.

Λύση

Ξεκινώ από την κορυφή C και διαγράφω τη πλευρά (C, Ε). Στη συνέχεια απο την κορυφή Ε διαγράφω την πλευρά (Ε, Α). Από την κορυφή Α επιλέγω την πλευρά (Α, Β) την οποία και δια­ γράφω. Στη συνέχεια, διαγράφω τις πλευρές με την εξής σειρά : (Β, D), (D, Α), (A,C), (C, F), (F, D) και τέλος την (D, C) . Ο κλειστός δρομίσκος που σχηματίστηκε είναι ο : C, Ε, Α, Β, D, Α, C, F, D, C. Χ1 :

Χ2:

Προτεινόμενες ασκήσεις

Να βρείτε ποια από τα παρακάτω γραφήματα είναι γράφημα-Όιλερ και ποια περιλαμβάνουν δρομίσκους:οιλερ :

Να βρείτε το κλειστό δρομίσκο στο γράφημα:οιλερ, χρησιμοποιώντας τον αλγόριθμο του Fleury :

ΕΥΚΛΕΙΔΗΣ Α' 86 τ.2/47


== ==

ΜΔ7.

Ο

Σπύρος Γεωρyiου

νατοί συνδυασμοί μετρήθηκαν και καταγρά­ όπως παρακάτω: Βρείτε έναν εξαψήφιο αριθμό του οποίου φηκαν 124, 1 3 1 , 137, 145, 1 5 1 , 1 53, 158, 1 59, 166, 180 το πρώτο ψηφίο είναι μία μονάδα μεγαλύτερο από το τρίτο, το δεύτερο ψηφίο είναι μία μο­ Μπορείτε χρησιμοποιώντας τις παραπάνω νάδα μικρότερο από το τέταρτο, το πέμπτο μετρήσεις, να βρείτε το βάρος του μεσαίου ψηφίο είναι μία μονάδα μικρότερο από το τρί­ ατόμου; (αν τους είχαμε ταξινομήσει κατά αύ­ το και το έκτο ψηφίο είναι μία μονάδα μεγα­ ξουσα σειρά βάρους ) λύτερο από το τέταρτο. Το άθροισμα του δεύτερου και του τρίτου ΜΔlΟ. Το τετράγωνο ψηφίου ισούται με το πρώτο, ενώ το άθροισμα Συνδέστε 4 κουκίδες στο παρακάτω πλέγ­ όλων των ψηφίων είναι 30. μα, έτσι ώστε να σχηματίσετε ένα τετράγωνο που να έχει εμβαδόν 8. ΜΔ8. Τα επτά ψηφία εξαψήφιος αριθμός

Τοποθετήστε τα επτά ψηφία παρακάτω στα κενά τετράγωνα, έτσι ώστε να αληθεύει το αποτέλεσμα της πρόσθεσης. Κάθε ψηφίο χρησιμοποιείται μία μόνο φορά.

[i][l] + DD@JD DDDD

ο, 1' 2, 3, 5, 8, 9

MΔl l. Πρώτοι αριθμοί

ΜΔ9. Τα βάρη

Βρείτε το άθροισμα του μικρότερου πρώ­ Σε μια προσπάθεια να κρατηθούν μυστικά του διψήφιου και του μεγαλύτερου πρώτου τα βάρη μιας ομάδας 5 ατόμων, ανέβαιναν και ζυγίζονταν κάθε φορά ανά δύο και όλοι οι δυ- διψήφιου αριθμού, των οποίων τα ψηφία είναι επίσης πρώτοι αριθμοί. ΕΥΚΛΕΙΔΗΣ Α'

86 τ.2/48


Α

��

Απαντήσεις προηγούμενου τεύχους

ΜΔ1 . Παιχνίδια με αριθμούς Έστω α και β είναι τα ψηφία του αριθμού χ, δηλαδή χ= 1 Οα+β. Άρα ο αριθμός y είναι ίσος με y= 1 0α+β-( α+β)=9α. Επομένως ο z=9α:9=α, δηλαδή ο z είναι ίσος με το πρώτο ψηφίο του διψήφιου αριθμού χ. Οι δυνατές τιμές του z είναι λοιπόν από το 1 έως το 9, και άρα ο μέ­ σος όρος των δυνατών τιμών του z είναι 5 . ΜΔ2. Στο αυτοκίνητο Φυσικά ο νέος παλινδρομικός αριθμός δεν μπορεί να αρχίζει από 2 ή 3 κ.λ.π. γιατί τότε π.χ. αν ήταν ο 23932 το αυτοκίνητο θα είχε δι­ ανύσει σε 2 ώρες 1 000 1 χιλιόμετρα! Άρα 2 λογικοί αριθμοί είναι ο 1 404 1 και 1 4 1 4 1 . Στην περίπτωση του πρώτου το αυτοκίνητο διένυσε 1 1 Ο Κm και άρα έτρεχε με μέση ταχύ­ τητα 55 Κm/h, ενώ στη δεύτερη περίπτωση διένυσε 2 1 Ο Κm και άρα έτρεχε με μέση ταχύ­ τητα 1 05Κm/h. Επομένως σύμφωνα με τους περιορισμούς, η λύση είναι ο 1 4 1 4 1 . ΜΔ3. Ο ι κάλτσες Πρέπει να πάρει 40 κάλτσες! Στη χειρότερη περίπτωση που πάρει όλες τις μπλε και όλες τις κόκκινες θα έχει τραβήξει συνολικά 38 κάλτσες. Έτσι αν πάρει ακόμα 2 θα έχει σίγουρα και ένα ζευγάρι από μαύρες κάλτσες. ΜΔ4. Πρώτοι αριθμοί Οι πρώτοι διψήφιοι πρώτοι είναι οι παρακάτω: 1 1 23 3 1 41 53 6 1 71 97 1 3 29 37 43 59 67 73 17

47

79

f

Jff

Άρα διαγράφουμε τους υπόλοιπους και μένουν οι εξής αριθμοί: 23 4 1 53 6 1 29 43 5 9 67 47 Από αυτούς με παρατήρηση έχουμε τις παρα­ κάτω τετράδες αριθμών: 23, 4 1 , 59 και 67 29, 47, 53 και 6 1 23, 47, 59 και 6 1 29, 4 1 , 5 3 και 67 Παρατηρείστε όμως ότι το άθροισμα οποιου­ δήποτε από τους παραπάνω συνδυασμούς εί­ ναι το ίδιο, δηλαδή : 23+4 1 +59+67=1 90 29+47+53+6 1 = 1 90 23+47+59+6 1 = 1 90 29+4 1 +53+67= 1 90 αφού τα 4 πρώτα ψηφία είναι πάντα ίδια και τα 4 δεύτερα ψηφία είναι επίσης πάντα ίδια. Άρα η απάντηση στο πρόβλημα είναι 1 90.

ΜΔS. Τα τετρ άγωνα Τα τετράγωνα σχηματίζονται ως εξής: -

Ο κάτω δεξιά όρος είναι το άθροισμα του πάνω αριστερά και του πάνω δεξιά όρου. - Ο κάτω αριστερά όρος είναι το διπλάσιο του κάτω αριστερά όρου. - Ο κεντρικός όρος είναι το τριπλάσιο του κάτω αριστερά όρου. Άρα ο αριθμός που λείπει είναι ο 3 • 1 0=30

19 Από αυτούς το 1 1 απορρίπτεται αμέσως γιατί κάθε ψηφίο χρησιμοποιείται μόνο μια φορά. Επίσης επειδή τα ψηφία 2, 4, 5, και 6 πρέπει να χρησιμοποιηθούν οπωσδήποτε μία φορά, θα πρέπει οι 4 διψήφιοι πρώτοι να αρχίζουν από αυτά τα ψηφία.

ΜΔ6. Ο κύβος Φέρνοντας τη τρίτη διαγώνιο AC σχηματίζεται ένα ισόπλευρο τρίγωνο ABC, αφού όλες οι δι­ αγώνιες των εδρών του κύβου είναι ίσες. Άρα η γωνία ABC είναι ίση με τη γωνία ισό­ ° πλευρου τριγώνου, δηλαδή 60 .

ΕΥΚΛΕΙΔΗΣ Α' 86 τ.2/49


ΑΠΟ ΦΑΣΗ ΤΟΥ Δ . Σ . ΤΗΣ Ε . Μ . Ε . Στη συνεδρίαση του Δ.Σ. της Ε.Μ.Ε. την 19η Οκτωβρίου 2012 και σύμφωνα με την ημερήσια διάταξη, συζητήθηκε η ομόφωνη πρόταση που κατέθεσε η 9μελής Επιτροπή που είχε οριστεί με ομόφωνη απόφαση της Γενικής Συνέλευσης της 1 1 ης Μαρτίου 201 2. Το Δ.Σ. αποφάσισε ομόφωνα να αποδεχθεί την πρόταση, όπως αυτή έχει λάβει την τελική μορφή της μετά από τη νομοτεχνική επεξεργασία και να τη θέσει υπόψη της Καταστατικής Γενικής Συνέλευσης στις 20 Ιανουαρίου 2013.

ΕΛΛΗΝΙΚΉ ΜΑΘΗΜΑΤΙΚΉ ΕΤΑΙΡΕΙΑ ------

ΠΡΟΣΚΛΗΣΗ

------

Αθήνα, 1 9 - 1 0 - 20 1 2 '

Καλούνται όλα τα τακτικά μέλη της Ε.Μ.Ε. σε

'

ΚΑΤΑΣΤΑΤΙΚΗ Γενική Συνέλευση την Κυριακή ΖΟ Ιανουαρίου 2013 ώρα 1 1.00 το πρωί, στο Μεγάλο Αμφιθέατρο στο Κτήριο του Νέου Χημείου του Πανεπιστημίου Αθηνών (Ναυαρίνου 13α, Αθήνα). Με μοναδικό Θέμα:

Τροποποίηση Καταστατικού

Για να υπάρξει απαρτία σύμφωνα με το καταστατικό πρέπει να παρίσταται τουλάχιστον το

1/2 των τακτικών μελών της Ε.Μ.Ε., που έχουν εκπλη ρώσει τις ταμειακές τους υποχρεώσεις

για το 20 1 2 .

Μ ε συναδελφικούς χαιρετισμούς Για το Δ. Σ. της Ελληνικής Μαθη ματικής Εταιρείας Ο Πρόεδρος

Ο Γενικός Γραμματέας

Γρηγόριος Καλογερόπουλος

Εμμανουήλ Κρητικός

Καθηγητής Πανεπιστη μίου Α θηνών

Λέκτορας Οικονομικού Πανεπιστημίου Αθηνών

Η συντακτική επιτροπή του ΕΥΚΛΕΙΔΗ Α' εύχεται σε όλους τους αναγνώστες

ΚΑΛΗ κaι ΔΗΜΙΟΥΡΓΙΚΗ' XPONIAI Με χαρά περιμένουμε νέες συνεργασίες, άρθρα, λύσεις ασκήσεων αλλά και τις παρατηρήσεις σας για να γίνουμε καλύτεροι . . .


Issuu converts static files into: digital portfolios, online yearbooks, online catalogs, digital photo albums and more. Sign up and create your flipbook.